Acc 304 midterm exam

Page 1

ACC 304 Week 5 Midterm Exam – Strayer NEW Click On The Link Below to Purchase A+ Graded Material Instant Download http://www.hwgala.com/ACC-304-Week-5-Midterm-Exam-Strayer-NEW-ACC304W5E.htm Week 5 Midterm Exam: Chapters 8 Through 11 CHAPTER 8 VALUATION OF INVENTORIES: A COST-BASIS APPROACH IFRS questions are available at the end of this chapter. TRUE FALSE—Conceptual 1.

A manufacturing concern would report the cost of units only partially processed as inventory in the balance sheet.

2.

Both merchandising and manufacturing companies normally have multiple inventory accounts.

3.

When using a perpetual inventory system, freight charges on goods purchased are debited to Freight-In.

4.

If a supplier ships goods f.o.b. destination, title passes to the buyer when the supplier delivers the goods to the common carrier.

5.

If ending inventory is understated, then net income is understated.

6.

If both purchases and ending inventory are overstated by the same amount, net income is not affected.

7.

Freight charges on goods purchased are considered a period cost and therefore are not part of the cost of the inventory.

8.

Purchase Discounts Lost is a financial expense and is reported in the “other expenses and losses” section of the income statement.

9.

The cost flow assumption adopted must be consistent with the physical movement of the goods.

10.

In all cases when FIFO is used, the cost of goods sold would be the same whether a perpetual or periodic system is used.


8-2

Test Bank for Intermediate Accounting, Fourteenth Edition

11.

The change in the LIFO Reserve from one period to the next is recorded as an adjustment to Cost of Goods Sold.

12.

Many companies use LIFO for both tax and internal reporting purposes.

13.

LIFO liquidation often distorts net income, but usually leads to substantial tax savings.

14.

LIFO liquidations can occur frequently when using a specific-goods approach.

15.

Dollar-value LIFO techniques help protect LIFO layers from erosion.

16.

The dollar-value LIFO method measures any increases and decreases in a pool in terms of total dollar value and physical quantity of the goods.

17.

A disadvantage of LIFO is that it does not match more recent costs against current revenues as well as FIFO.

18.

The LIFO conformity rule requires that if a company uses LIFO for tax purposes, it must also use LIFO for financial accounting purposes.

19.

Use of LIFO provides a tax benefit in an industry where unit costs tend to decrease as production increases.

20.

LIFO is inappropriate where unit costs tend to decrease as production increases.

True False Answers—Conceptual

MULTIPLE CHOICE—Conceptual 21.

Which of the following inventories carried by a manufacturer is similar to the merchandise inventory of a retailer? a. Raw materials. b. Work-in-process. c. Finished goods. d. Supplies.

22.

Where should raw materials be classified on the balance sheet? a. Prepaid expenses. b. Inventory. c. Equipment. d. Not on the balance sheet.

23.

Which of the following accounts is not reported in inventory? a. Raw materials.


Valuation of Inventories: A Cost-Basis Approach

8-3

b. Equipment. c. Finished goods. d. Supplies. 24.

Why are inventories included in the computation of net income? a. To determine cost of goods sold. b. To determine sales revenue. c. To determine merchandise returns. d. Inventories are not included in the computation of net income.

25.

Which of the following is a characteristic of a perpetual inventory system? a. Inventory purchases are debited to a Purchases account. b. Inventory records are not kept for every item. c. Cost of goods sold is recorded with each sale. d. Cost of goods sold is determined as the amount of purchases less the change in inventory.


8-4

Test Bank for Intermediate Accounting, Fourteenth Edition

26.

How is a significant amount of consignment inventory reported in the balance sheet? a. The inventory is reported separately on the consignor's balance sheet. b. The inventory is combined with other inventory on the consignor's balance sheet. c. The inventory is reported separately on the consignee's balance sheet. d. The inventory is combined with other inventory on the consignee's balance sheet.

27.

Where should goods in transit that were recently purchased f.o.b. destination be included on the balance sheet? a. Accounts payable. b. Inventory. c. Equipment. d. Not on the balance sheet.

28.

If a company uses the periodic inventory system, what is the impact on net income of including goods in transit f.o.b. shipping point in purchases, but not ending inventory? a. Overstate net income. b. Understate net income. c. No effect on net income. d. Not sufficient information to determine effect on net income.

29.

If a company uses the periodic inventory system, what is the impact on the current ratio of including goods in transit f.o.b. shipping point in purchases, but not ending inventory? a. Overstate the current ratio. b. Understate the current ratio. c. No effect on the current ratio. d. Not sufficient information to determine effect on the current ratio.

30.

What is consigned inventory? a. Goods that are shipped, but title transfers to the receiver. b. Goods that are sold, but payment is not required until the goods are sold. c. Goods that are shipped, but title remains with the shipper. d. Goods that have been segregated for shipment to a customer.

31.

When using a perpetual inventory system, a. no Purchases account is used. b. a Cost of Goods Sold account is used. c. two entries are required to record a sale. d. all of these.

32.

Goods in transit which are shipped f.o.b. shipping point should be a. included in the inventory of the seller. b. included in the inventory of the buyer. c. included in the inventory of the shipping company. d. none of these.

33.

Goods in transit which are shipped f.o.b. destination should be a. included in the inventory of the seller.


Valuation of Inventories: A Cost-Basis Approach

8-5

b. included in the inventory of the buyer. c. included in the inventory of the shipping company. d. none of these. 34.

Which of the following items should be included in a company's inventory at the balance sheet date? a. Goods in transit which were purchased f.o.b. destination. b. Goods received from another company for sale on consignment. c. Goods sold to a customer which are being held for the customer to call for at his or her convenience. d. None of these.

Use the following information for questions 35 and 36. During 2012 Carne Corporation transferred inventory to Nolan Corporation and agreed to repurchase the merchandise early in 2013. Nolan then used the inventory as collateral to borrow from Norwalk Bank, remitting the proceeds to Carne. In 2013 when Carne repurchased the inventory, Nolan used the proceeds to repay its bank loan. 35.

This transaction is known as a(n) a. consignment. b. installment sale. c. assignment for the benefit of creditors. d. product financing arrangement.

36.

On whose books should the cost of the inventory appear at the December 31, 2012 balance sheet date? a. Carne Corporation b. Nolan Corporation c. Norwalk Bank d. Nolan Corporation, with Carne making appropriate note disclosure of the transaction

37.

Goods on consignment are a. included in the consignee's inventory. b. recorded in a Consignment Out account which is an inventory account. c. recorded in a Consignment In account which is an inventory account. d. all of these

S

Valuation of inventories requires the determination of all of the following except a. the costs to be included in inventory. b. the physical goods to be included in inventory. c. the cost of goods held on consignment from other companies. d. the cost flow assumption to be adopted.

P

The accountant for the Pryor Sales Company is preparing the income statement for 2012 and the balance sheet at December 31, 2012. Pryor uses the periodic inventory system. The January 1, 2012 merchandise inventory balance will appear a. only as an asset on the balance sheet.

38.

39.


8-6

Test Bank for Intermediate Accounting, Fourteenth Edition

b. only in the cost of goods sold section of the income statement. c. as a deduction in the cost of goods sold section of the income statement and as a current asset on the balance sheet. d. as an addition in the cost of goods sold section of the income statement and as a current asset on the balance sheet.


Valuation of Inventories: A Cost-Basis Approach

8-7

P

If the beginning inventory for 2012 is overstated, the effects of this error on cost of goods sold for 2012, net income for 2012, and assets at December 31, 2013, respectively, are a. overstatement, understatement, overstatement. b. overstatement, understatement, no effect. c. understatement, overstatement, overstatement. d. understatement, overstatement, no effect.

S

41.

The failure to record a purchase of merchandise on account even though the goods are properly included in the physical inventory results in a. an overstatement of assets and net income. b. an understatement of assets and net income. c. an understatement of cost of goods sold and liabilities and an overstatement of assets. d. an understatement of liabilities and an overstatement of owners' equity.

42.

Dolan Co. received merchandise on consignment. As of March 31, Dolan had recorded the transaction as a purchase and included the goods in inventory. The effect of this on its financial statements for March 31 would be a. no effect. b. net income was correct and current assets and current liabilities were overstated. c. net income, current assets, and current liabilities were overstated. d. net income and current liabilities were overstated.

43.

Green Co. received merchandise on consignment. As of January 31, Green included the goods in inventory, but did not record the transaction. The effect of this on its financial statements for January 31 would be a. net income, current assets, and retained earnings were overstated. b. net income was correct and current assets were understated. c. net income and current assets were overstated and current liabilities were understated. d. net income, current assets, and retained earnings were understated.

44.

Feine Co. accepted delivery of merchandise which it purchased on account. As of December 31, Feine had recorded the transaction, but did not include the merchandise in its inventory. The effect of this on its financial statements for December 31 would be a. net income, current assets, and retained earnings were understated. b. net income was correct and current assets were understated. c. net income was understated and current liabilities were overstated. d. net income was overstated and current assets were understated.

45.

On June 15, 2012, Wynne Corporation accepted delivery of merchandise which it purchased on account. As of June 30, Wynne had not recorded the transaction or included the merchandise in its inventory. The effect of this on its balance sheet for June 30, 2012 would be a. assets and stockholders' equity were overstated but liabilities were not affected. b. stockholders' equity was the only item affected by the omission. c. assets, liabilities, and stockholders' equity were understated. d. none of these.

40.


8-8

Test Bank for Intermediate Accounting, Fourteenth Edition

46.

What is the effect of a $50,000 overstatement of last year's inventory on current years ending retained earning balance? a. Understated by $50,000. b. No effect. c. Overstated by $50,000. d. Need more information to determine.

47.

Which of the following is a product cost as it relates to inventory? a. Selling costs. b. Interest costs. c. Raw materials. d. Abnormal spoilage.

48.

Which of the following is a period cost? a. Labor costs. b. Freight in. c. Production costs. d. Selling costs.

49.

Which method may be used to record cash discounts a company receives for paying suppliers promptly? a. Net method. b. Gross method. c. Average method. d. a and b.

50.

Which of the following is included in inventory costs? a. Product costs. b. Period costs. c. Product and period costs. d. Neither product or period costs.

51.

Which of the following is correct? a. Selling costs are product costs. b. Manufacturing overhead costs are product costs. c. Interest costs for routine inventories are product costs. d. All of these.

52.

All of the following costs should be charged against revenue in the period in which costs are incurred except for a. manufacturing overhead costs for a product manufactured and sold in the same accounting period. b. costs which will not benefit any future period. c. costs from idle manufacturing capacity resulting from an unexpected plant shutdown. d. costs of normal shrinkage and scrap incurred for the manufacture of a product in ending inventory.


Valuation of Inventories: A Cost-Basis Approach

8-9

53.

Which of the following types of interest cost incurred in connection with the purchase or manufacture of inventory should be capitalized as a product cost? a. Purchase discounts lost b. Interest incurred during the production of discrete projects such as ships or real estate projects c. Interest incurred on notes payable to vendors for routine purchases made on a repetitive basis d. All of these should be capitalized.

54.

The use of a Discounts Lost account implies that the recorded cost of a purchased inventory item is its a. invoice price. b. invoice price plus the purchase discount lost. c. invoice price less the purchase discount taken. d. invoice price less the purchase discount allowable whether taken or not.

55.

The use of a Purchase Discounts account implies that the recorded cost of a purchased inventory item is its a. invoice price. b. invoice price plus any purchase discount lost. c. invoice price less the purchase discount taken. d. invoice price less the purchase discount allowable whether taken or not.

Use the following information for questions 56 and 57. During 2012, which was the first year of operations, Oswald Company had merchandise purchases of $985,000 before cash discounts. All purchases were made on terms of 2/10, n/30. Threefourths of the items purchased were paid for within 10 days of purchase. All of the goods available had been sold at year end. 56.

Which of the following recording procedures would result in the highest cost of goods sold for 2012? 1. Recording purchases at gross amounts 2. Recording purchases at net amounts, with the amount of discounts not taken shown under "other expenses" in the income statement a. 1 b. 2 c. Either 1 or 2 will result in the same cost of goods sold. d. Cannot be determined from the information provided.

57.

Which of the following recording procedures would result in the highest net income for 2012? 1. Recording purchases at gross amounts 2. Recording purchases at net amounts, with the amount of discounts not taken shown under "other expenses" in the income statement a. 1 b. 2


8 - 10

Test Bank for Intermediate Accounting, Fourteenth Edition

c. Either 1 or 2 will result in the same net income. d. Cannot be determined from the information provided.


Valuation of Inventories: A Cost-Basis Approach

58.

8 - 11

When using the periodic inventory system, which of the following generally would not be separately accounted for in the computation of cost of goods sold? a. Trade discounts applicable to purchases during the period b. Cash (purchase) discounts taken during the period c. Purchase returns and allowances of merchandise during the period d. Cost of transportation-in for merchandise purchased during the period

S

Costs which are inventoriable include all of the following except a. costs that are directly connected with the bringing of goods to the place of business of the buyer. b. costs that are directly connected with the converting of goods to a salable condition. c. buying costs of a purchasing department. d. selling costs of a sales department.

P

60.

Which inventory costing method most closely approximates current cost for each of the following: Ending Inventory Cost of Goods Sold a. FIFO FIFO b. FIFO LIFO c. LIFO FIFO d. LIFO LIFO

61.

In situations where there is a rapid turnover, an inventory method which produces a balance sheet valuation similar to the first-in, first-out method is a. average cost. b. base stock. c. joint cost. d. prime cost.

62.

The pricing of issues from inventory must be deferred until the end of the accounting period under the following method of inventory valuation: a. moving average. b. weighted-average. c. LIFO perpetual. d. FIFO.

63.

An inventory pricing procedure in which the oldest costs incurred rarely have an effect on the ending inventory valuation is a. FIFO. b. LIFO. c. base stock. d. weighted-average.

64.

Which method of inventory pricing best approximates specific identification of the actual flow of costs and units in most manufacturing situations? a. Average cost b. First-in, first-out

59.


8 - 12

Test Bank for Intermediate Accounting, Fourteenth Edition

c. Last-in, first-out d. Base stock 65.

Assuming no beginning inventory, what can be said about the trend of inventory prices if cost of goods sold computed when inventory is valued using the FIFO method exceeds cost of goods sold when inventory is valued using the LIFO method? a. Prices decreased. b. Prices remained unchanged. c. Prices increased. d. Price trend cannot be determined from information given.

66.

In a period of rising prices, the inventory method which tends to give the highest reported net income is a. base stock. b. first-in, first-out. c. last-in, first-out. d. weighted-average.

67.

In a period of rising prices, the inventory method which tends to give the highest reported inventory is a. FIFO. b. moving average. c. LIFO. d. weighted-average.

68.

Tanner Corporation's inventory cost on its balance sheet was lower using first-in, first-out than it would have been using last-in, first-out. Assuming no beginning inventory, in what direction did the cost of purchases move during the period? a. Up b. Down c. Steady d. Cannot be determined

69.

In a period of rising prices, the inventory method which tends to give the highest reported cost of goods sold is a. FIFO. b. average cost. c. LIFO. d. none of these.

70.

Which of the following statements is not valid as it applies to inventory costing methods? a. If inventory quantities are to be maintained, part of the earnings must be invested (plowed back) in inventories when FIFO is used during a period of rising prices. b. LIFO tends to smooth out the net income pattern by matching current cost of goods sold with current revenue, when inventories remain at constant quantities.


Valuation of Inventories: A Cost-Basis Approach

8 - 13

c. When a firm using the LIFO method fails to maintain its usual inventory position (reduces stock on hand below customary levels), there may be a matching of old costs with current revenue. d. The use of FIFO permits some control by management over the amount of net income for a period through controlled purchases, which is not true with LIFO.


8 - 14

Test Bank for Intermediate Accounting, Fourteenth Edition

71.

The acquisition cost of a certain raw material changes frequently. The book value of the inventory of this material at year end will be the same if perpetual records are kept as it would be under a periodic inventory method only if the book value is computed under the a. weighted-average method. b. moving average method. c. LIFO method. d. FIFO method.

72.

Which of the following is a reason why the specific identification method may be considered ideal for assigning costs to inventory and cost of goods sold? a. The potential for manipulation of net income is reduced. b. There is no arbitrary allocation of costs. c. The cost flow matches the physical flow. d. Able to use on all types of inventory.

73.

In a period of rising prices which inventory method generally provides the greatest amount of net income? a. Average cost. b. FIFO. c. LIFO. d. Specific identification.

74.

In a period of falling prices, which inventory method generally provides the greatest amount of net income? a. Average cost. b. FIFO. c. LIFO. d. Specific identification.

75.

What is a LIFO reserve? a. The difference between the LIFO inventory and the amount used for internal reporting purposes. b. The tax savings attributed to using the LIFO method. c. The current effect of using LIFO on net income. d. Change in the LIFO inventory during the year.

76.

When a company uses LIFO for external reporting purposes and FIFO for internal reporting purposes, an Allowance to Reduce Inventory to LIFO account is used. This account should be reported a. on the income statement in the Other Revenues and Gains section. b. on the income statement in the Cost of Goods Sold section. c. on the income statement in the Other Expenses and Losses section. d. on the balance sheet in the Current Assets section.

77.

What happens when inventory in base year dollars decreases? a. LIFO reserve increases. b. LIFO layer is created.


Valuation of Inventories: A Cost-Basis Approach

c. LIFO layer is liquidated. d. LIFO price index decreases.

8 - 15


8 - 16

Test Bank for Intermediate Accounting, Fourteenth Edition

78.

How might a company obtain a price index in order to apply dollar-value LIFO? a. Calculate an index based on recent inventory purchases. b. Use a general price level index published by the government. c. Use a price index prepared by an industry group. d. All of the above.

79.

In the context of dollar-value LIFO, what is a LIFO layer? a. The difference between the LIFO inventory and the amount used for internal reporting purposes. b. The LIFO value of the inventory for a given year. c. The inventory in base year dollars. d. The LIFO value of an increase in the inventory for a given year.

S

Which of the following statements is not true as it relates to the dollar-value LIFO inventory method? a. It is easier to erode LIFO layers using dollar-value LIFO techniques than it is with specific goods pooled LIFO. b. Under the dollar-value LIFO method, it is possible to have the entire inventory in only one pool. c. Several pools are commonly employed in using the dollar-value LIFO inventory method. d. Under dollar-value LIFO, increases and decreases in a pool are determined and measured in terms of total dollar value, not physical quantity.

S

81.

Which of the following is not considered an advantage of LIFO when prices are rising? a. The inventory will be overstated. b. The more recent costs are matched against current revenues. c. There will be a deferral of income tax. d. A company's future reported earnings will not be affected substantially by future price declines.

82.

Which of the following is true regarding the use of LIFO for inventory valuation? a. If LIFO is used for external financial reporting, then it must also be used for internal reports. b. For purposes of external financial reporting, LIFO may not be used with the lower of cost or market approach. c. If LIFO is used for external financial reporting, then it cannot be used for tax purposes. d. None of these.

83.

If inventory levels are stable or increasing, an argument which is not an advantage of the LIFO method as compared to FIFO is a. income taxes tend to be reduced in periods of rising prices. b. cost of goods sold tends to be stated at approximately current cost on the income statement. c. cost assignments typically parallel the physical flow of goods. d. income tends to be smoothed as prices change over time.

80.


Valuation of Inventories: A Cost-Basis Approach

8 - 17


8 - 18

Test Bank for Intermediate Accounting, Fourteenth Edition

Multiple Choice Answers—Conceptual

MULTIPLE CHOICE—Computational 84.

Morgan Manufacturing Company has the following account balances at year end: Office supplies Raw materials Work-in-process Finished goods Prepaid insurance

$ 4,000 27,000 59,000 82,000 6,000

What amount should Morgan report as inventories in its balance sheet? a. $82,000. b. $86,000. c. $168,000. d. $172,000. 85.

Lawson Manufacturing Company has the following account balances at year end: Office supplies Raw materials Work-in-process Finished goods Prepaid insurance

$ 4,000 27,000 59,000 97,000 6,000

What amount should Lawson report as inventories in its balance sheet? a. $97,000. b. $101,000. c. $183,000. d. $187,000.


Valuation of Inventories: A Cost-Basis Approach

8 - 19

86.

Elkins Corporation uses the perpetual inventory method. On March 1, it purchased $20,000 of inventory, terms 2/10, n/30. On March 3, Elkins returned goods that cost $2,000. On March 9, Elkins paid the supplier. On March 9, Elkins should credit a. purchase discounts for $400. b. inventory for $400. c. purchase discounts for $360. d. inventory for $360.

87.

Malone Corporation uses the perpetual inventory method. On March 1, it purchased $50,000 of inventory, terms 2/10, n/30. On March 3, Malone returned goods that cost $5,000. On March 9, Malone paid the supplier. On March 9, Malone should credit a. purchase discounts for $1,000. b. inventory for $1,000. c. purchase discounts for $900. d. inventory for $900.

88.

Bell Inc. took a physical inventory at the end of the year and determined that $780,000 of goods were on hand. In addition, Bell, Inc. determined that $60,000 of goods that were in transit that were shipped f.o.b. shipping point were actually received two days after the inventory count and that the company had $90,000 of goods out on consignment. What amount should Bell report as inventory at the end of the year? a. $780,000. b. $840,000. c. $870,000. d. $930,000.

89.

Bell Inc. took a physical inventory at the end of the year and determined that $760,000 of goods were on hand. In addition, the following items were not included in the physical count. Bell, Inc. determined that $96,000 of goods were in transit that were shipped f.o.b. destination (goods were actually received by the company three days after the inventory count).The company sold $40,000 worth of inventory f.o.b. destination. What amount should Bell report as inventory at the end of the year? a. $760,000. b. $856,000. c. $800,000. d. $896,000.

90.

Risers Inc. reported total assets of $1,800,000 and net income of $200,000 for the current year. Risers determined that inventory was overstated by $15,000 at the beginning of the year (this was not corrected). What is the corrected amount for total assets and net income for the year? a. $1,800,000 and $200,000. b. $1,800,000 and $215,000. c. $1,785,000 and $185,000. d. $1,815,000 and $215,000.


8 - 20

91.

Test Bank for Intermediate Accounting, Fourteenth Edition

Risers Inc. reported total assets of $3,200,000 and net income of $170,000 for the current year. Risers determined that inventory was understated by $46,000 at the beginning of the year and $20,000 at the end of the year. What is the corrected amount for total assets and net income for the year? a. $3,220,000 and $190,000. b. $3,180,000 and $196,000. c. $3,220,000 and $144,000. d. $3,200,000 and $170,000.

Use the following information for questions 92 through 94. Hudson, Inc. is a calendar-year corporation. Its financial statements for the years 2013 and 2012 contained errors as follows: 2013 2012 Ending inventory $4,500 overstated $12,000 overstated Depreciation expense $3,000 understated $9,000 overstated 92.

Assume that the proper correcting entries were made at December 31, 2012. By how much will 2013 income before taxes be overstated or understated? a. $1,500 understated b. $1,500 overstated c. $3,000 overstated d. $7,500 overstated

93.

Assume that no correcting entries were made at December 31, 2012. Ignoring income taxes, by how much will retained earnings at December 31, 2013 be overstated or understated? a. $1,500 understated b. $7,500 overstated c. $7,500 understated d. $13,500 understated

94.

Assume that no correcting entries were made at December 31, 2012, or December 31, 2013 and that no additional errors occurred in 2014. Ignoring income taxes, by how much will working capital at December 31, 2014 be overstated or understated? a. $0 b. $3,000 overstated c. $3,000 understated d. $7,500 understated

95.

The following information is available for Naab Company for 2012: Freight-in Purchase returns Selling expenses Ending inventory

$ 30,000 75,000 200,000 260,000


Valuation of Inventories: A Cost-Basis Approach

8 - 21

The cost of goods sold is equal to 400% of selling expenses. What is the cost of goods available for sale? a. $800,000. b. $1,090,000. c. $1,015,000. d. $1,060,000. Use the following information for questions 96 and 97. Winsor Co. records purchases at net amounts. On May 5 Winsor purchased merchandise on account, $20,000, terms 2/10, n/30. Winsor returned $1,500 of the May 5 purchase and received credit on account. At May 31 the balance had not been paid. 96.

The amount to be recorded as a purchase return is a. $1,350. b. $1,530 c. $1,500. d. $1,470.

97.

By how much should the account payable be adjusted on May 31? a. $0. b. $430. c. $400. d. $370.

Use the following information for questions 98 and 99. The following information was available from the inventory records of Rich Company for January: Balance at January 1 Purchases: January 6 January 26 Sales: January 7 January 31 Balance at January 31 98.

Units 3,000

Unit Cost $9.77

2,000 2,700

10.30 10.71

Total Cost $29,310 20,600 28,917

(2,500) (4,300) 900

Assuming that Rich does not maintain perpetual inventory records, what should be the inventory at January 31, using the weighted-average inventory method, rounded to the nearest dollar? a. $9,454. b. $9,213. c. $9,234. d. $9,324.


8 - 22

99.

Test Bank for Intermediate Accounting, Fourteenth Edition

Assuming that Rich maintains perpetual inventory records, what should be the inventory at January 31, using the moving-average inventory method, rounded to the nearest dollar? a. $9,454. b. $9,213. c. $9,234. d. $9,324.


Valuation of Inventories: A Cost-Basis Approach

8 - 23

Use the following information for questions 100 and 101. Niles Co. has the following data related to an item of inventory: Inventory, March 1 100 units @ $2.10 Purchase, March 7 350 units @ $2.20 Purchase, March 16 70 units @ $2.25 Inventory, March 31 130 units 100.

The value assigned to ending inventory if Niles uses LIFO is a. $290. b. $276. c. $273. d. $292.

101.

The value assigned to cost of goods sold if Niles uses FIFO is a. $290. b. $276. c. $862. d. $848.

102.

Emley Company has been using the LIFO method of inventory valuation for 10 years, since it began operations. Its 2012 ending inventory was $60,000, but it would have been $90,000 if FIFO had been used. Thus, if FIFO had been used, Emley's income before income taxes would have been a. $30,000 greater over the 10-year period. b. $30,000 less over the 10-year period. c. $30,000 greater in 2012. d. $30,000 less in 2012.

Use the following information for questions 103 through 106. Transactions for the month of June were: Purchases June 1 (balance) 1,200 @ $3.20 3 3,300 @ 3.10 7 1,800 @ 3.30 15 2,700 @ 3.40 22 750 @ 3.50 103.

June 2 6 9 10 18 25

Sales 900 @ $5.50 2,400 @ 5.50 1,500 @ 5.50 600 @ 6.00 2,100 @ 6.00 300 @ 6.00

Assuming that perpetual inventory records are kept in units only, the ending inventory on a LIFO basis is a. $6,165. b. $6,240. c. $6,435. d. $6,705.


8 - 24

104.

105.

Test Bank for Intermediate Accounting, Fourteenth Edition

Assuming that perpetual inventory records are kept in dollars, the ending inventory on a LIFO basis is a. $6,165. b. $6,240. c. $6,435. d. $6,705. Assuming that perpetual inventory records are kept in dollars, the ending inventory on a FIFO basis is a. $6,165. b. $6,240. c. $6,435. d. $6,705.

106.

Assuming that perpetual inventory records are kept in units only, the ending inventory on an average-cost basis, rounded to the nearest dollar, is a. $6,144. b. $6,357. c. $6,435. d. $6,483.

107.

Milford Company had 500 units of “Tank” in its inventory at a cost of $4 each. It purchased, for $2,800, 300 more units of “Tank”. Milford then sold 400 units at a selling price of $10 each, resulting in a gross profit of $1,600. The cost flow assumption used by Johnson a. is FIFO. b. is LIFO. c. is weighted average. d. cannot be determined from the information given.

108.

Nichols Company had 500 units of “Dink” in its inventory at a cost of $5 each. It purchased, for $2,400, 300 more units of “Dink”. Nichols then sold 600 units at a selling price of $10 each, resulting in a gross profit of $2,100. The cost flow assumption used by Nichols. a. is FIFO. b. is LIFO. c. is weighted average. d. cannot be determined from the information given.

109.

June Corp. sells one product and uses a perpetual inventory system. The beginning inventory consisted of 20 units that cost $20 per unit. During the current month, the company purchased 120 units at $20 each. Sales during the month totaled 90 units for $43 each. What is the number of units in the ending inventory? a. 20 units. b. 30 units. c. 50 units. d. 140 units.


Valuation of Inventories: A Cost-Basis Approach

110.

8 - 25

June Corp. sells one product and uses a perpetual inventory system. The beginning inventory consisted of 20 units that cost $20 per unit. During the current month, the company purchased 120 units at $20 each. Sales during the month totaled 90 units for $43 each. What is the cost of goods sold using the LIFO method? a. $400. b. $1,800. c. $2,400. d. $3,870.


8 - 26

Test Bank for Intermediate Accounting, Fourteenth Edition

111.

Checkers uses the periodic inventory system. For the current month, the beginning inventory consisted of 2,400 units that cost $12 each. During the month, the company made two purchases: 1,000 units at $13 each and 4,000 units at $13.50 each. Checkers also sold 4,300 units during the month. Using the average cost method, what is the amount of cost of goods sold for the month? a. $55,685. b. $57,900. c. $53,950. d. $55,900.

112.

Chess Top uses the periodic inventory system. For the current month, the beginning inventory consisted of 300 units that cost $65 each. During the month, the company made two purchases: 450 units at $68 each and 225 units at $70 each. Chess Top also sold 750 units during the month. Using the average cost method, what is the amount of ending inventory? a. $15,750. b. $50,655. c. $50,100. d. $15,197.

113.

Checkers uses the periodic inventory system. For the current month, the beginning inventory consisted of 2,400 units that cost $12 each. During the month, the company made two purchases: 1,000 units at $13 each and 4,000 units at $13.50 each. Checkers also sold 4,300 units during the month. Using the FIFO method, what is the ending inventory? a. $40,146. b. $37,200. c. $41,850. d. $37,900.

114.

Chess Top uses the periodic inventory system. For the current month, the beginning inventory consisted of 300 units that cost $65 each. During the month, the company made two purchases: 450 units at $68 each and 225 units at $70 each. Chess Top also sold 750 units during the month. Using the FIFO method, what is the amount of cost of goods sold for the month? a. $50,655. b. $48,750. c. $51,225. d. $50,100.

115.

Checkers uses the periodic inventory system. For the current month, the beginning inventory consisted of 2,400 units that cost $12 each. During the month, the company made two purchases: 1,000 units at $13 each and 4,000 units at $13.50 each. Checkers also sold 4,300 units during the month. Using the LIFO method, what is the ending inventory? a. $40,146. b. $37,200. c. $41,850. d. $37,900.


Valuation of Inventories: A Cost-Basis Approach

8 - 27


8 - 28

Test Bank for Intermediate Accounting, Fourteenth Edition

116.

Chess Top uses the periodic inventory system. For the current month, the beginning inventory consisted of 300 units that cost $65 each. During the month, the company made two purchases: 450 units at $68 each and 225 units at $70 each. Chess Top also sold 750 units during the month. Using the LIFO method, what is the amount of cost of goods sold for the month? a. $50,655. b. $48,750. c. $51,225. d. $50,100.

117.

Black Corporation uses the FIFO method for internal reporting purposes and LIFO for external reporting purposes. The balance in the LIFO Reserve account at the end of 2012 was $100,000. The balance in the same account at the end of 2013 is $150,000. Black’s Cost of Goods Sold account has a balance of $750,000 from sales transactions recorded during the year. What amount should Black report as Cost of Goods Sold in the 2013 income statement? a. $700,000. b. $750,000. c. $800,000. d. $900,000.

118.

White Corporation uses the FIFO method for internal reporting purposes and LIFO for external reporting purposes. The balance in the LIFO Reserve account at the end of 2012 was $120,000. The balance in the same account at the end of 2013 is $180,000. White’s Cost of Goods Sold account has a balance of $900,000 from sales transactions recorded during the year. What amount should White report as Cost of Goods Sold in the 2013 income statement? a. $840,000. b. $900,000. c. $960,000. d. $1,080,000.

119.

Milford Company had 400 units of “Tank” in its inventory at a cost of $8 each. It purchased 600 more units of “Tank” at a cost of $12 each. Milford then sold 700 units at a selling price of $20 each. The LIFO liquidation overstated normal gross profit by a. $ -0b. $400. c. $800. d. $1,200.

120.

Nichols Company had 400 units of “Dink” in its inventory at a cost of $10 each. It purchased 600 more units of “Dink” at a cost of $15 each. Nichols then sold 700 units at a selling price of $25 each. The LIFO liquidation overstated normal gross profit by a. $ -0b. $500. c. $1,000. d. $1,500.


Valuation of Inventories: A Cost-Basis Approach

8 - 29


8 - 30

Test Bank for Intermediate Accounting, Fourteenth Edition

Use the following information for 121 and 122 RF Company had January 1 inventory of $150,000 when it adopted dollar-value LIFO. During the year, purchases were $900,000 and sales were $1,500,000. December 31 inventory at year-end prices was $215,040, and the price index was 112. 121.

What is RF Company’s ending inventory? a. $150,000. b. $192,000. c. $197,040. d. $215,040.

122.

What is RF Company’s gross profit? a. $642,000. b. $647,040. c. $665,190. d. $1,302,960.

Use the following information for 123 and 124 Hay Company had January 1 inventory of $120,000 when it adopted dollar-value LIFO. During the year, purchases were $720,000 and sales were $1,200,000. December 31 inventory at year-end prices was $151,800, and the price index was 110. 123.

What is Hay Company’s ending inventory? a. $132,000. b. $138,000. c. $139,800. d. $151,800.

124.

What is Hay Company’s gross profit? a. $498,000. b. $499,800. c. $511,800. d. $1,060,200.

Use the following information for questions 125 through 127. Gross Corporation adopted the dollar-value LIFO method of inventory valuation on December 31, 2011. Its inventory at that date was $440,000 and the relevant price index was 100. Information regarding inventory for subsequent years is as follows: Date December 31, 2012 December 31, 2013 December 31, 2014

Inventory at Current Prices $513,600 580,000 650,000

Current Price Index 107 125 130


Valuation of Inventories: A Cost-Basis Approach

125.

126.

8 - 31

What is the cost of the ending inventory at December 31, 2012 under dollar-value LIFO? a. $480,000. b. $513,600. c. $482,800. d. $470,800. What is the cost of the ending inventory at December 31, 2013 under dollar-value LIFO? a. $464,000. b. $462,800. c. $465,680. d. $480,000.

127.

What is the cost of the ending inventory at December 31, 2014 under dollar-value LIFO? a. $512,480. b. $509,600. c. $500,000. d. $526,800.

128.

Wise Company adopted the dollar-value LIFO method on January 1, 2012, at which time its inventory consisted of 6,000 units of Item A @ $5.00 each and 3,000 units of Item B @ $16.00 each. The inventory at December 31, 2012 consisted of 12,000 units of Item A and 7,000 units of Item B. The most recent actual purchases related to these items were as follows: Quantity Items Purchase Date Purchased Cost Per Unit A 12/7/12 2,000 $ 6.00 A 12/11/12 10,000 5.75 B 12/15/12 7,000 17.00 Using the double-extension method, what is the price index for 2012 that should be computed by Wise Company? a. 108.33% b. 109.59% c. 111.05% d. 220.51%

129.

Web World began using dollar-value LIFO for costing its inventory last year. The base year layer consists of $350,000. Assuming the current inventory at end of year prices equals $483,000 and the index for the current year is 1.10, what is the ending inventory using dollar-value LIFO? a. $483,000. b. $448,000. c. $439,091. d. $531,300.

130.

Willy World began using dollar-value LIFO for costing its inventory two years ago. The ending inventory for the past two years in end-of-year dollars was $120,000 and $180,000 and the year-end price indices were 1.0 and 1.2, respectively. Assuming the current


8 - 32

Test Bank for Intermediate Accounting, Fourteenth Edition

inventory at end of year prices equals $258,000 and the index for the current year is 1.25, what is the ending inventory using dollar-value LIFO? a. $213,000. b. $223,680. c. $228,000. d. $226,500.


Valuation of Inventories: A Cost-Basis Approach

131.

Opera Corp. uses the dollar-value LIFO method of computing its inventory cost. Data for the past four years is as follows: Year ended Inventory at December 31. End-of-year Prices 2011 $130,000 2012 252,000 2013 270,000 What is the 2011 inventory balance using dollar-value LIFO? a. $130,000. b. $123,808. c. $245,454. d. $270,000.

132.

Price Index 1.00 1.05 1.10

Opera Corp. uses dollar-value LIFO method of computing its inventory cost. Data for the past four years is as follows: Year ended Inventory at December 31. End-of-year Prices 2011 $ 130,000 2012 252,000 2013 270,000 What is the 2012 inventory balance using dollar-value LIFO? a. $252,000. b. $257,000. c. $245,500. d. $251,500.

133.

8 - 33

Price Index 1.00 1.05 1.10

Opera Corp. uses dollar-value LIFO method of computing its inventory cost. Data for the past four years is as follows: Year ended Inventory at December 31. End-of-year Prices 2011 $ 130,000 2012 252,000 2013 270,000 What is the 2013 inventory balance using dollar-value LIFO? a. $270,000. b. $257,000. c. $245,500. d. $251,500.

Price Index 1.00 1.05 1.10


Test Bank for Intermediate Accounting, Fourteenth Edition

8 - 34

Multiple Choice Answers—Computational

MULTIPLE CHOICE—CPA Adapted 134.

How should the following costs affect a retailer's inventory valuation? a. b. c. d.

135.

Freight-in Increase Increase No effect No effect

Interest on Inventory Loan No effect Increase Increase No effect

The following information applied to Howe, Inc. for 2012: Merchandise purchased for resale $350,000 Freight-in 8,000 Freight-out 5,000 Purchase returns 2,000 Howe's 2012 inventoriable cost was a. $350,000. b. $353,000. c. $356,000. d. $361,000.

136.

The following information was derived from the 2012 accounting records of Perez Co.: Perez 's Goods Perez 's Central Warehouse Held by Consignees Beginning inventory $130,000 $ 14,000 Purchases 475,000 70,000 Freight-in 10,000 Transportation to consignees 5,000 Freight-out 30,000 8,000 Ending inventory 145,000 20,000


Valuation of Inventories: A Cost-Basis Approach

8 - 35

Perez's 2012 cost of sales was a. $470,000. b. $500,000. c. $534,000. d. $539,000. 137.

Dole Corp.'s accounts payable at December 31, 2012, totaled $650,000 before any necessary year-end adjustments relating to the following transactions: On December 27, 2012, Dole wrote and recorded checks to creditors totaling $350,000 causing an overdraft of $100,000 in Dole's bank account at December 31, 2012. The checks were mailed out on January 10, 2013. •

On December 28, 2012, Dole purchased and received goods for $150,000, terms 2/10, n/30. Dole records purchases and accounts payable at net amounts. The invoice was recorded and paid January 3, 2013. •

Goods shipped f.o.b. destination on December 20, 2012 from a vendor to Dole were received January 2, 2013. The invoice cost was $65,000. •

At December 31, 2012, what amount should Dole report as total accounts payable? a. $1,212,000. b. $1,147,000. c. $900,000. d. $800,000. 138.

The balance in Moon Co.'s accounts payable account at December 31, 2012 was $900,000 before any necessary year-end adjustments relating to the following: Goods were in transit to Moon from a vendor on December 31, 2012. The invoice cost was $40,000. The goods were shipped f.o.b. shipping point on December 29, 2012 and were received on January 4, 2013. •

Goods shipped f.o.b. destination on December 21, 2012 from a vendor to Moon were received on January 6, 2013. The invoice cost was $25,000. •

On December 27, 2012, Moon wrote and recorded checks to creditors totaling $30,000 that were mailed on January 10, 2013. •

In Moon's December 31, 2012 balance sheet, the accounts payable should be a. $930,000. b. $940,000. c. $965,000. d. $970,000. 139.

Kerr Co.'s accounts payable balance at December 31, 2012 was $1,300,000 before considering the following transactions: Goods were in transit from a vendor to Kerr on December 31, 2012. The invoice price was $70,000, and the goods were shipped f.o.b. shipping point on December 29, 2012. The goods were received on January 4, 2013. •


8 - 36

Test Bank for Intermediate Accounting, Fourteenth Edition

Goods shipped to Kerr, f.o.b. shipping point on December 20, 2012, from a vendor were lost in transit. The invoice price was $50,000. On January 5, 2013, Kerr filed a $50,000 claim against the common carrier. •


Valuation of Inventories: A Cost-Basis Approach

8 - 37

In its December 31, 2012 balance sheet, Kerr should report accounts payable of a. $1,420,000. b. $1,370,000. c. $1,350,000. d. $1,300,000. 140.

Walsh Retailers purchased merchandise with a list price of $75,000, subject to trade discounts of 20% and 10%, with no cash discounts allowable. Walsh should record the cost of this merchandise as a. $52,500. b. $54,000. c. $58,500. d. $75,000.

141.

On June 1, 2012, Penny Corp. sold merchandise with a list price of $40,000 to Linn on account. Penny allowed trade discounts of 30% and 20%. Credit terms were 2/15, n/40 and the sale was made f.o.b. shipping point. Penny prepaid $800 of delivery costs for Ison as an accommodation. On June 12, 2012, Penny received from Linn a remittance in full payment amounting to a. $21,952. b. $22,736. c. $22,752. d. $22,392.

142.

Groh Co. recorded the following data pertaining to raw material X during January 2012: Units Date Received Cost Issued On Hand 1/1/12 Inventory $4.00 3,200 1/11/12 Issue 1,600 1,600 1/22/12 Purchase 4,000 $4.70 5,600 The moving-average unit cost of X inventory at January 31, 2012 is a. $4.35. b. $4.42. c. $4.50. d. $4.70.

143.

During periods of rising prices, a perpetual inventory system would result in the same dollar amount of ending inventory as a periodic inventory system under which of the following inventory cost flow methods? FIFO LIFO a. Yes No b. Yes Yes c. No Yes d. No No


Test Bank for Intermediate Accounting, Fourteenth Edition

8 - 38

144.

Hite Co. was formed on January 2, 2012, to sell a single product. Over a two-year period, Hite's acquisition costs have increased steadily. Physical quantities held in inventory were equal to three months' sales at December 31, 2012, and zero at December 31, 2013. Assuming the periodic inventory system, the inventory cost method which reports the highest amount of each of the following is Inventory Cost of Sales December 31, 2012 2013 a. LIFO FIFO b. LIFO LIFO c. FIFO FIFO d. FIFO LIFO

145.

Keck Co. had 450 units of product A on hand at January 1, 2012, costing $21 each. Purchases of product A during January were as follows: Date Units Unit Cost Jan. 10 600 $22 18 750 23 28 300 24 A physical count on January 31, 2012 shows 600 units of product A on hand. The cost of the inventory at January 31, 2012 under the LIFO method is a. $14,100. b. $13,350. c. $12,750. d. $12,300.

146.

When the double extension approach to the dollar-value LIFO inventory cost flow method is used, the inventory layer added in the current year is multiplied by an index number. How would the following be used in the calculation of this index number?

a. b. c. d. 147.

Ending inventory at current year cost Numerator Numerator Denominator Not used

Ending inventory at base year cost Denominator Not used Numerator Denominator

Farr Co. adopted the dollar-value LIFO inventory method on December 31, 2012. Farr's entire inventory constitutes a single pool. On December 31, 2012, the inventory was $480,000 under the dollar-value LIFO method. Inventory data for 2013 are as follows: 12/31/13 inventory at year-end prices Relevant price index at year end (base year 2012)

$660,000 110

Using dollar value LIFO, Farr's inventory at December 31, 2013 is a. $528,000. b. $612,000. c. $600,000. d. $660,000.


Valuation of Inventories: A Cost-Basis Approach

8 - 39


8 - 40

Test Bank for Intermediate Accounting, Fourteenth Edition

Multiple Choice Answers—CPA Adapted CHAPTER 9 INVENTORIES: ADDITIONAL VALUATION ISSUES IFRS questions are available at the end of this chapter. TRUE-FALSE—Conceptual 1.

A company should abandon the historical cost principle when the future utility of the inventory item falls below its original cost.

2.

The lower-of-cost-or-market method is used for inventory despite being less conservative than valuing inventory at market value.

3.

The purpose of the “floor” in lower-of-cost-or-market considerations is to avoid overstating inventory.

4.

Application of the lower-of-cost-or-market rule results in inconsistency because a company may value inventory at cost in one year and at market in the next year.

5.

GAAP requires reporting inventory at net realizable value, even if above cost, whenever there is a controlled market with a quoted price applicable to all quantities.

6.

A reason for valuing inventory at net realizable value is that sometimes it is too difficult to obtain the cost figures.

7.

In a basket purchase, the cost of the individual assets acquired is determined on the basis of their relative sales value.

8.

A basket purchase occurs when a company agrees to buy inventory weeks or months in advance.

9.

Most purchase commitments must be recorded as a liability.

10.

If the contract price on a noncancelable purchase commitment exceeds the market price, the buyer should record any expected losses on the commitment in the period in which the market decline takes place.

11.

When a buyer enters into a formal, noncancelable purchase contract, an asset and a liability are recorded at the inception of the contract.

12.

The gross profit method can be used to approximate the dollar amount of inventory on hand.

13.

In most situations, the gross profit percentage is stated as a percentage of cost.


Valuation of Inventories: A Cost-Basis Approach

8 - 41

14.

A disadvantage of the gross profit method is that it uses past percentages in determining the markup.

15.

When the conventional retail method includes both net markups and net markdowns in the cost-to-retail ratio, it approximates a lower-of-cost-or-market valuation.

16.

In the retail inventory method, the term markup means a markup on the original cost of an inventory item.

17.

In the retail inventory method, abnormal shortages are deducted from both the cost and retail amounts and reported as a loss. The inventory turnover ratio is computed by dividing the cost of goods sold by the ending inventory on hand.

18. 19.

The average days to sell inventory represents the average number of days’ sales for which a company has inventory on hand.

*20.

The LIFO retail method assumes that markups and markdowns apply only to the goods purchased during the period.

True False Answers—Conceptual MULTIPLE CHOICE—Conceptual 21. a.

Which of the following is true about lower-of-cost-or-market? It is inconsistent because losses are recognized but not gains.

b.

It usually understates assets.

c.

It can increase future income.

d.

All of these.

22. a.

The primary basis of accounting for inventories is cost. A departure from the cost basis of pricing the inventory is required where there is evidence that when the goods are sold in the ordinary course of business their selling price will be less than their replacement cost.

b.

replacement cost will be more than their net realizable value.

c.

cost will be less than their replacement cost.


8 - 42

d.

23.

Test Bank for Intermediate Accounting, Fourteenth Edition

future utility will be less than their cost.

a.

When valuing raw materials inventory at lower-of-cost-or-market, what is the meaning of the term "market"? Net realizable value

b.

Net realizable value less a normal profit margin

c.

Current replacement cost

d.

Discounted present value

24. a.

In no case can "market" in the lower-of-cost-or-market rule be more than estimated selling price in the ordinary course of business.

b.

estimated selling price in the ordinary course of business less reasonably predictable costs

of completion and disposal. c.

estimated selling price in the ordinary course of business less reasonably predictable costs

of completion and disposal and an allowance for an approximately normal profit margin. d.

estimated selling price in the ordinary course of business less reasonably predictable costs

of completion and disposal, an allowance for an approximately normal profit margin, and an adequate reserve for possible future losses. 25.

Designated market value a. is always the middle value of replacement cost, net realizable value, and net realizable value less a normal profit margin. b. should always be equal to net realizable value. c. may sometimes exceed net realizable value. d. should always be equal to net realizable value less a normal profit margin.

26. a.

Lower-of-cost-or-market is most conservative if applied to the total inventory.

b.

is most conservative if applied to major categories of inventory.


Valuation of Inventories: A Cost-Basis Approach

c.

is most conservative if applied to individual items of inventory.

d.

must be applied to major categories for taxes.

27.

8 - 43

a.

An item of inventory purchased this period for $15.00 has been incorrectly written down to its current replacement cost of $10.00. It sells during the following period for $30.00, its normal selling price, with disposal costs of $3.00 and normal profit of $12.00. Which of the following statements is not true? The cost of sales of the following year will be understated.

b.

The current year's income is understated.

c.

The closing inventory of the current year is understated.

d.

Income of the following year will be understated.

S

28. a.

When the cost-of-goods-sold method is used to record inventory at market there is a direct reduction in the selling price of the product that results in a loss being

recorded on the income statement prior to the sale. b.

a loss is recorded directly in the inventory account by crediting inventory and debiting loss

on inventory decline. c.

only the portion of the loss attributable to inventory sold during the period is recorded in

the financial statements. d.

the market value figure for ending inventory is substituted for cost and the loss is buried in

cost of goods sold.

29.

Lower-of-cost-or-market as it applies to inventory is best described as the a. drop of future utility below its original cost. b. method of determining cost of goods sold. c. assumption to determine inventory flow. d. change in inventory value to market value.


8 - 44

Test Bank for Intermediate Accounting, Fourteenth Edition

30.

The floor to be used in applying the lower-of-cost-or-market method to inventory is determined as the a. net realizable value. b. net realizable value less normal profit margin. c. replacement cost. d. selling price less costs of completion and disposal.

31.

What is the rationale behind the ceiling when applying the lower-of-cost-or-market method to inventory? a. Prevents understatement of the inventory value. b. Allows for a normal profit to be earned. c. Allows for items to be valued at replacement cost. d. Prevents overstatement of the value of obsolete or damaged inventories.

32.

Why are inventories stated at lower-of-cost-or-market? a. To report a loss when there is a decrease in the future utility. b. To be conservative. c. To report a loss when there is a decrease in the future utility below the original cost. d. To permit future profits to be recognized.

33.

Which of the following is not an acceptable approach in applying the lower-of-cost-ormarket method to inventory? a. Inventory location. b. Categories of inventory items. c. Individual item. d. Total of the inventory.

34.

Which method(s) may be used to record a loss due to a price decline in the value of inventory? a. Cost-of-goods-sold. b. Sales method. c. Loss method d. Both a and c.

35.

Why might inventory be reported at sales prices (net realizable value or market price) rather than cost? a. When there is a controlled market with a quoted price applicable to all quantities and when there are no significant costs of disposal. b. When there are no significant costs of disposal. c. When a non-cancellable contract exists to sell the inventory. d. When there is a controlled market with a quoted price applicable to all quantities.

S

36.

a.

Recording inventory at net realizable value is permitted, even if it is above cost, when there are no significant costs of disposal involved and the ending inventory is determined by a physical inventory count.

b.

a normal profit is not anticipated.


Valuation of Inventories: A Cost-Basis Approach

8 - 45

c.

there is a controlled market with a quoted price applicable to all quantities.

d.

the internal revenue service is assured that the practice is not used only to distort reported

net income.

37. a.

When inventory declines in value below original (historical) cost, and this decline is considered other than temporary, what is the maximum amount that the inventory can be valued at? Sales price

b.

Net realizable value

c.

Historical cost

d.

Net realizable value reduced by a normal profit margin


8 - 46

Test Bank for Intermediate Accounting, Fourteenth Edition

38. a.

Net realizable value is acquisition cost plus costs to complete and sell.

b.

selling price.

c.

selling price plus costs to complete and sell.

d.

selling price less costs to complete and sell.

39. a.

If a unit of inventory has declined in value below original cost, but the market value exceeds net realizable value, the amount to be used for purposes of inventory valuation is net realizable value.

b.

original cost.

c.

market value.

d.

net realizable value less a normal profit margin.

40. a.

Inventory may be recorded at net realizable value if there is a controlled market with a quoted price.

b.

there are no significant costs of disposal.

c.

the inventory consists of precious metals or agricultural products.

d.

all of these.

41. a.

If a material amount of inventory has been ordered through a formal purchase contract at the balance sheet date for future delivery at firm prices, this fact must be disclosed.

b.

disclosure is required only if prices have declined since the date of the order.

c.

disclosure is required only if prices have since risen substantially.

d.

an appropriation of retained earnings is necessary.


Valuation of Inventories: A Cost-Basis Approach

42.

8 - 47

a.

The credit balance that arises when a net loss on a purchase commitment is recognized should be presented as a current liability.

b.

subtracted from ending inventory.

c.

presented as an appropriation of retained earnings.

d.

presented in the income statement.

P

43.

a.

In 2012, Orear Manufacturing signed a contract with a supplier to purchase raw materials in 2013 for $700,000. Before the December 31, 2012 balance sheet date, the market price for these materials dropped to $510,000. The journal entry to record this situation at December 31, 2012 will result in a credit that should be reported as a valuation account to Inventory on the balance sheet.

b.

as a current liability.

c.

as an appropriation of retained earnings.

d.

on the income statement.

44.

At the end of the fiscal year, Apha Airlines has an outstanding non-cancellable purchase commitment for the purchase of 1 million gallons of jet fuel at a price of $4.10 per gallon for delivery during the coming summer. The company prices its inventory at the lower of cost or market. If the market price for jet fuel at the end of the year is $4.50, how would this situation be reflected in the annual financial statements? a. Record unrealized gains of $400,000 and disclose the existence of the purchase commitment. b. No impact. c. Record unrealized losses of $400,000 and disclose the existence of the purchase commitment. d. Disclose the existence of the purchase commitment.

45.

At the end of the fiscal year, Apha Airlines has an outstanding purchase commitment for the purchase of 1 million gallons of jet fuel at a price of $4.60 per gallon for delivery during the coming summer. The company prices its inventory at the lower of cost or market. If the market price for jet fuel at the end of the year is $4.25, how would this situation be reflected in the annual financial statements? a. Record unrealized gains of $350,000 and disclose the existence of the purchase commitment. b. No impact. c. Record unrealized losses of $350,000 and disclose the existence of the purchase commitment. d. Disclose the existence of the purchase commitment.


8 - 48

46.

S

Test Bank for Intermediate Accounting, Fourteenth Edition

How is the gross profit method used as it relates to inventory valuation? a. Verify the accuracy of the perpetual inventory records. b. Verity the accuracy of the physical inventory. c. To estimate cost of goods sold. d. To provide an inventory value of LIFO inventories.

47. a.

Which of the following is not a basic assumption of the gross profit method? The beginning inventory plus the purchases equal total goods to be accounted for.

b.

Goods not sold must be on hand.

c.

If the sales, reduced to the cost basis, are deducted from the sum of the opening inventory

plus purchases, the result is the amount of inventory on hand. d.

The total amount of purchases and the total amount of sales remain relatively unchanged

from the comparable previous period.

48. a.

The gross profit method of inventory valuation is invalid when a portion of the inventory is destroyed.

b.

there is a substantial increase in inventory during the year.

c.

there is no beginning inventory because it is the first year of operation.

d.

none of these.

49. a.

Which statement is not true about the gross profit method of inventory valuation? It may be used to estimate inventories for interim statements.

b.

It may be used to estimate inventories for annual statements.

c.

It may be used by auditors.

d.

None of these.

50.

A major advantage of the retail inventory method is that it


Valuation of Inventories: A Cost-Basis Approach

a.

8 - 49

provides reliable results in cases where the distribution of items in the inventory is

different from that of items sold during the period. b.

hides costs from competitors and customers.

c.

gives a more accurate statement of inventory costs than other methods.

d.

provides a method for inventory control and facilitates determination of the periodic

inventory for certain types of companies.

51. a.

An inventory method which is designed to approximate inventory valuation at the lower of cost or market is last-in, first-out.

b.

first-in, first-out.

c.

conventional retail method.

d.

specific identification.


8 - 50

52. a.

Test Bank for Intermediate Accounting, Fourteenth Edition

The retail inventory method is based on the assumption that the final inventory and the total of goods available for sale contain the same proportion of

high-cost and low-cost ratio goods. b.

ratio of gross margin to sales is approximately the same each period.

c.

ratio of cost to retail changes at a constant rate.

d.

proportions of markups and markdowns to selling price are the same.

53. a.

Which statement is true about the retail inventory method? It may not be used to estimate inventories for interim statements.

b.

It may not be used to estimate inventories for annual statements.

c.

It may not be used by auditors.

d.

None of these.

54. a.

When the conventional retail inventory method is used, markdowns are commonly ignored in the computation of the cost to retail ratio because there may be no markdowns in a given year.

b.

this tends to give a better approximation of the lower of cost or market.

c.

markups are also ignored.

d.

this tends to result in the showing of a normal profit margin in a period when no markdown

goods have been sold.

55. a.

To produce an inventory valuation which approximates the lower of cost or market using the conventional retail inventory method, the computation of the ratio of cost to retail should include markups but not markdowns.

b.

include markups and markdowns.


Valuation of Inventories: A Cost-Basis Approach

8 - 51

c.

ignore both markups and markdowns.

d.

include markdowns but not markups.

*56. a.

When calculating the cost ratio for the retail inventory method, if it is the conventional method, the beginning inventory is included and markdowns are

deducted. b.

if it is the LIFO method, the beginning inventory is excluded and markdowns are deducted.

c.

if it is the LIFO method, the beginning inventory is included and markdowns are not

deducted. d.

if it is the conventional method, the beginning inventory is excluded and markdowns are

not deducted.

S

57. a.

Which of the following is not required when using the retail inventory method? All inventory items must be categorized according to the retail markup percentage which

reflects the item's selling price. b.

A record of the total cost and retail value of goods purchased.

c.

A record of the total cost and retail value of the goods available for sale.

d.

Total sales for the period.

S

58. a.

Which of the following is not a reason the retail inventory method is used widely? As a control measure in determining inventory shortages

b.

For insurance information

c.

To permit the computation of net income without a physical count of inventory

d.

To defer income tax liability


8 - 52

Test Bank for Intermediate Accounting, Fourteenth Edition

59.

What condition is not necessary in order to use the retail method to provide inventory results? a. Retailer keeps a record of the total costs of products sold for the period. b. Retailer keeps a record of the total costs and retail value of goods purchased. c. Retailer keeps a record of the total costs and retail value of goods available for sale. d. Retailer keeps a record of sales for the period.

60.

What method yields results that are essentially the same as those of the conventional retail method? a. FIFO. b. Lower-of-average-cost-or-market. c. Average cost. d. LIFO.

61.

What is the effect of net markups on the cost-retail ratio when using the conventional retail method? a. Increases the cost-retail ratio. b. No effect on the cost-retail ratio. c. Depends on the amount of the net markdowns. d. Decreases the cost-retail ratio.

62.

What is the effect of freight-in on the cost-retail ratio when using the conventional retail method? a. Increases the cost-retail ratio. b. No effect on the cost-retail ratio. c. Depends on the amount of the net markups. d. Decreases the cost-retail ratio.

63.

Which of the following is not a common disclosure for inventories? a. Inventory composition. b. Inventory location. c. Inventory financing arrangements. d. Inventory costing methods employed.

P

64. a.

Which of the following statements is false regarding an assumption of inventory cost flow? The cost flow assumption need not correspond to the actual physical flow of goods.

b.

The assumption selected may be changed each accounting period.

c.

The FIFO assumption uses the earliest acquired prices to cost the items sold during a

period.


Valuation of Inventories: A Cost-Basis Approach

d.

The LIFO assumption uses the earliest acquired prices to cost the items on hand at the end

of an accounting period.

P

8 - 53

65. a.

The average days to sell inventory is computed by dividing 365 days by the inventory turnover ratio.

b.

the inventory turnover ratio by 365 days.

c.

net sales by the inventory turnover ratio.

d.

365 days by cost of goods sold.


8 - 54

Test Bank for Intermediate Accounting, Fourteenth Edition

66. a.

The inventory turnover ratio is computed by dividing the cost of goods sold by beginning inventory.

b.

ending inventory.

c.

average inventory.

d.

number of days in the year.

*67. a.

When using dollar-value LIFO, if the incremental layer was added last year, it should be multiplied by last year's cost ratio and this year's index.

b.

this year's cost ratio and this year's index.

c.

last year's cost ratio and last year's index.

d.

this year's cost ratio and last year's index.

Multiple Choice Answers—Conceptual Solutions to those Multiple Choice questions for which the answer is “none of these.” 48.

The gross profit percentage applicable to the goods in ending inventory is different from the percentage applicable to the goods sold during the period.

53.

Many answers are possible.

MULTIPLE CHOICE—Computational 68.

Oslo Corporation has two products in its ending inventory, each accounted for at the lower of cost or market. A profit margin of 30% on selling price is considered normal for each product. Specific data with respect to each product follows: Historical cost Replacement cost Estimated cost to dispose Estimated selling price

Product #1 $20.00 22.50 5.00 40.00

Product #2 $ 35.00 27.00 13.00 65.00

In pricing its ending inventory using the lower-of-cost-or-market, what unit values should Oslo use for products #1 and #2, respectively?


Valuation of Inventories: A Cost-Basis Approach

a.

$20.00 and $32.50.

b.

$23.00 and $32.50.

c.

$23.00 and $30.00.

d.

$22.50 and $27.00.

69.

8 - 55

a.

Muckenthaler Company sells product 2005WSC for $30 per unit. The cost of one unit of 2005WSC is $27, and the replacement cost is $26. The estimated cost to dispose of a unit is $6, and the normal profit is 40%. At what amount per unit should product 2005WSC be reported, applying lower-of-cost-or-market? $12.

b.

$24.

c.

$26.

d.

$27.

70.

a.

Lexington Company sells product 1976NLC for $50 per unit. The cost of one unit of 1976NLC is $45, and the replacement cost is $43. The estimated cost to dispose of a unit is $10, and the normal profit is 40%. At what amount per unit should product 1976NLC be reported, applying lower-of-cost-or-market? $20.

b.

$40.

c.

$43.

d.

$45.

71.

Given the acquisition cost of product Z is $64, the net realizable value for product Z is $58, the normal profit for product Z is $5, and the market value (replacement cost) for product Z is $60, what is the proper per unit inventory price for product Z? a. $64. b. $60. c. $53. d. $58.


8 - 56

Test Bank for Intermediate Accounting, Fourteenth Edition

72.

Given the acquisition cost of product ALPHA is $17, the net realizable value for product ALPHA is $16.70, the normal profit for product ALPHA is $1.24, and the market value (replacement cost) for product ALPHA is $14.72, what is the proper per unit inventory price for product ALPHA? a. $17.00. b. $15.46 c. $14.72. d. $16.70.

73.

Given the acquisition cost of product Dominoe is $43.31, the net realizable value for product Dominoe is $38.49, the normal profit for product Dominoe is $4.32, and the market value (replacement cost) for product Dominoe is $40.68, what is the proper per unit inventory price for product Dominoe? a. $40.68. b. $34.18. c. $38.49. d. $43.31

74.

Given the historical cost of product Z is $80, the selling price of product Z is $95, costs to sell product Z are $11, the replacement cost for product Z is $83, and the normal profit margin is 40% of sales price, what is the market value that should be used in the lower-ofcost-or-market comparison? a. $80. b. $84. c. $83. d. $46. Given the historical cost of product Z is $80, the selling price of product Z is $95, costs to sell product Z are $11, the replacement cost for product Z is $83, and the normal profit margin is 40% of sales price, what is the amount that should be used to value the inventory under the lower-of-cost-or-market method? a. $46. b. $80. c. $84. d. $83.

75.

76.

Given the historical cost of product Dominoe is $43, the selling price of product Dominoe is $60, costs to sell product Dominoe are $11, the replacement cost for product Dominoe is $40, and the normal profit margin is 20% of sales price, what is the cost amount that should be used in the lower-of-cost-or-market comparison? a. $49. b. $40. c. $37. d. $43.

77.

Given the historical cost of product Dominoe is $43, the selling price of product Dominoe is $60, costs to sell product Dominoe are $11, the replacement cost for product Dominoe is


Valuation of Inventories: A Cost-Basis Approach

8 - 57

$40, and the normal profit margin is 20% of sales price, what is the amount that should be used to value the inventory under the lower-of-cost-or-market method? a. $43. b. $37. c. $40. d. $49. 78.

Robust Inc. has the following information related to an item in its ending inventory. Product 66 has a cost of $3,250, a replacement cost of $3,100, a net realizable value of $3,200, and a normal profit margin of $200. What is the final lower-of-cost-or-market inventory value for product 66? a. $3,200. b. $3,100. c. $3,250. d. $3,100.

79.

Robust Inc. has the following information related to an item in its ending inventory. Packit (Product # 874) has a cost of $524, a replacement cost of $402, a net realizable value of $468, and a normal profit margin of $21. What is the final lower-of-cost-or-market inventory value for Packit? a. $447. b. $524. c. $402. d. $468.


8 - 58

Test Bank for Intermediate Accounting, Fourteenth Edition

80.

Robust Inc. has the following information related to an item in its ending inventory. Acer Top has a cost of $251, a replacement cost of $234, a net realizable value of $266, and a normal profit margin of $34. What is the final lower-of-cost-or-market inventory value for Acer Top? a. $232. b. $251. c. $234. d. $266.

81.

a.

Mortenson Corporation sells its product, a rare metal, in a controlled market with a quoted price applicable to all quantities. The total cost of 5,000 pounds of the metal now held in inventory is $150,000. The total selling price is $360,000, and estimated costs of disposal are $10,000. At what amount should the inventory of 5,000 pounds be reported in the balance sheet? $140,000.

b.

$150,000.

c.

$350,000.

d.

$360,000.

82.

a.

Rodriguez Corporation sells its product, a rare metal, in a controlled market with a quoted price applicable to all quantities. The total cost of 5,000 pounds of the metal now held in inventory is $210,000. The total selling price is $490,000, and estimated costs of disposal are $5,000. At what amount should the inventory of 5,000 pounds be reported in the balance sheet? $205,000.

b.

$210,000.

c.

$485,000.

d.

$490,000.

83.

a.

Turner Corporation acquired two inventory items at a lump-sum cost of $80,000. The acquisition included 3,000 units of product LF, and 7,000 units of product 1B. LF normally sells for $24 per unit, and 1B for $8 per unit. If Turner sells 1,000 units of LF, what amount of gross profit should it recognize? $3,000


Valuation of Inventories: A Cost-Basis Approach

b.

$9,000.

c.

$16,000.

d.

$19,000.

84.

8 - 59

a.

Robertson Corporation acquired two inventory items at a lump-sum cost of $60,000. The acquisition included 3,000 units of product CF, and 7,000 units of product 3B. CF normally sells for $18 per unit, and 3B for $6 per unit. If Robertson sells 1,000 units of CF, what amount of gross profit should it recognize? $2,250.

b.

$6,750.

c.

$12,000.

d.

$14,250.


8 - 60

85.

Test Bank for Intermediate Accounting, Fourteenth Edition

At a lump-sum cost of $72,000, Pratt Company recently purchased the following items for resale: Item M N O

No. of Items Purchased 4,000 2,000 6,000

Resale Price Per Unit $3.75 12.00 6.00

The appropriate cost per unit of inventory is: M N O a. $3.75 $12.00 $6.00 b. $3.11 $19.86 $3.32 c. $3.60 $11.52 $5.76 d. $6.00 $6.00 $6.00 86.

Confectioners, a chain of candy stores, purchases its candy in bulk from its suppliers. For a recent shipment, the company paid $1,800 and received 8,500 pieces of candy that are allocated among three groups. Group 1 consists of 2,500 pieces that are expected to sell for $0.15 each. Group 2 consists of 5,500 pieces that are expected to sell for $0.36 each. Group 3 consists of 500 pieces that are expected to sell for $0.72 each. Using the relative sales value method, what is the cost per item in Group 1? a. $0.150. b. $0.100. c. $0.120. d. $0.225.

87.

Confectioners, a chain of candy stores, purchases its candy in bulk from its suppliers. For a recent shipment, the company paid $1,800 and received 8,500 pieces of candy that are allocated among three groups. Group 1 consists of 2,500 pieces that are expected to sell for $0.15 each. Group 2 consists of 5,500 pieces that are expected to sell for $0.36 each. Group 3 consists of 500 pieces that are expected to sell for $0.72 each. Using the relative sales value method, what is the cost per item in Group 2? a. $0.225. b. $0.360. c. $0.210. d. $0.239.

88.

Confectioners, a chain of candy stores, purchases its candy in bulk from its suppliers. For a recent shipment, the company paid $1,800 and received 8,500 pieces of candy that are allocated among three groups. Group 1 consists of 2,500 pieces that are expected to sell for $0.15 each. Group 2 consists of 5,500 pieces that are expected to sell for $0.36 each. Group 3 consists of 500 pieces that are expected to sell for $0.72 each. Using the relative sales value method, what is the cost per item in Group 3? a. $0.477. b. $0.225. c. $0.720. d. $0.540.


Valuation of Inventories: A Cost-Basis Approach

8 - 61

89.

During the current fiscal year, Jeremiah Corp. signed a long-term noncancellable purchase commitment with its primary supplier. Jeremiah agreed to purchase $2.5 million of raw materials during the next fiscal year under this contract. At the end of the current fiscal year, the raw material to be purchased under this contract had a market value of $2.3 million. What is the journal entry at the end of the current fiscal year? a. Debit Unrealized Holding Gain or Loss for $200,000 and credit Estimated Liability on Purchase Commitment for $200,000. b. Debit Estimated liability on Purchase Commitments for $200,000 and credit Unrealized Holding Gain or Loss for $200,000. c. Debit Unrealized Holding Gain or Loss for $2,300,000 and credit Estimated Liability on Purchase Commitments for $2,300,000. d. No journal entry is required.

90.

During the prior fiscal year, Jeremiah Corp. signed a long-term noncancellable purchase commitment with its primary supplier to purchase $2.5 million of raw materials. Jeremiah paid the $2.5 million to acquire the raw materials when the raw materials were only worth $2.3 million. Assume that the purchase commitment was properly recorded. What is the journal entry to record the purchase? a. Debit Inventory for $2,300,000, and credit Cash for $2,300,000. b. Debit Inventory for $2,300,000, debit Unrealized Holding Gain or Loss for $200,000, and credit Cash for $2,500,000. c. Debit Inventory for $2,300,000, debit Estimated Liability on Purchase Commitments for $200,000 and credit Cash for $2,500,000. d. Debit Inventory for $2,500,000, and credit Cash for $2,500,000.

91.

During 2012, Larue Co., a manufacturer of chocolate candies, contracted to purchase 200,000 pounds of cocoa beans at $4.00 per pound, delivery to be made in the spring of 2013. Because a record harvest is predicted for 2013, the price per pound for cocoa beans had fallen to $3.30 by December 31, 2012. Of the following journal entries, the one which would properly reflect in 2012 the effect of the commitment of Larue Co. to purchase the 100,000 pounds of cocoa is a. Cocoa Inventory............................................................... 400,000 Accounts Payable................................................. 400,000 b. Cocoa Inventory............................................................... 330,000 Loss on Purchase Commitments...................................... 70,000 Accounts Payable................................................. 400,000 c. Unrealized Holding Gain or Loss-Income....................... 70,000 Estimated Liability on Purchase Commitments... 70,000 d. No entry would be necessary in 2012

92.

a.

RS Corporation, a manufacturer of ethnic foods, contracted in 2012 to purchase 500 pounds of a spice mixture at $5.00 per pound, delivery to be made in spring of 2013. By 12/31/12, the price per pound of the spice mixture had risen to $5.40 per pound. In 2012, AJ should recognize a loss of $2,500.


8 - 62

Test Bank for Intermediate Accounting, Fourteenth Edition

b.

a loss of $200.

c.

no gain or loss.

d.

a gain of $200.


Valuation of Inventories: A Cost-Basis Approach

93.

8 - 63

a

LF Corporation, a manufacturer of Mexican foods, contracted in 2012 to purchase 1,000 pounds of a spice mixture at $5.00 per pound, delivery to be made in spring of 2013. By 12/31/12, the price per pound of the spice mixture had dropped to $4.70 per pound. In 2012, LF should recognize a loss of $5,000.

b.

a loss of $300.

c.

no gain or loss.

d.

a gain of $300.

94.

The following information is available for October for Barton Company. Beginning inventory Net purchases Net sales Percentage markup on cost

$150,000 450,000 900,000 66.67%

a.

A fire destroyed Barton’s October 31 inventory, leaving undamaged inventory with a cost of $9,000. Using the gross profit method, the estimated ending inventory destroyed by fire is $51,000.

b.

$231,000.

c.

$240,000.

d.

$300,000.

95.

The following information is available for October for Norton Company. Beginning inventory Net purchases Net sales Percentage markup on cost

$200,000 600,000 1,200,000 66.67%

a.

A fire destroyed Norton’s October 31 inventory, leaving undamaged inventory with a cost of $12,000. Using the gross profit method, the estimated ending inventory destroyed by fire is $68,000.

b.

$308,000.


8 - 64

Test Bank for Intermediate Accounting, Fourteenth Edition

c.

$320,000.

d.

$400,000.

Use the following information for questions 96 and 97. Miles Company, a wholesaler, budgeted the following sales for the indicated months: Sales on account Cash sales Total sales

June $2,700,000 270,000 $2,970,000

July $2,760,000 300,000 $3,060,000

August $2,850,000 390,000 $3,240,000

All merchandise is marked up to sell at its invoice cost plus 20%. Merchandise inventories at the beginning of each month are at 30% of that month's projected cost of goods sold.


Valuation of Inventories: A Cost-Basis Approach

96. a.

The cost of goods sold for the month of June is anticipated to be $2,160,000.

b.

$2,250,000.

c.

$2,280,000.

d.

$2,475,000.

97. a.

Merchandise purchases for July are anticipated to be $2,448,000.

b.

$3,114,000.

c.

$2,550,000.

d.

$2,595,000.

98.

8 - 65

a.

Reyes Company had a gross profit of $480,000, total purchases of $560,000, and an ending inventory of $320,000 in its first year of operations as a retailer. Reyes’s sales in its first year must have been $720,000.

b.

$880,000.

c.

$240,000.

d.

$800,000.

99. a.

A markup of 30% on cost is equivalent to what markup on selling price? 23%

b.

30%

c.

70%

d.

77%


8 - 66

100.

Test Bank for Intermediate Accounting, Fourteenth Edition

Kesler, Inc. estimates the cost of its physical inventory at March 31 for use in an interim financial statement. The rate of markup on cost is 25%. The following account balances are available: Inventory, March 1 Purchases Purchase returns Sales during March

$385,000 301,000 14,000 525,000

a.

The estimate of the cost of inventory at March 31 would be $147,000.

b.

$252,000.

c.

$278,250.

d.

$196,000.

101.

a.

On January 1, 2012, the merchandise inventory of Glaus, Inc. was $1,000,000. During 2012 Glaus purchased $2,000,000 of merchandise and recorded sales of $2,500,000. The gross profit rate on these sales was 25%. What is the merchandise inventory of Glaus at December 31, 2012? $500,000.

b.

$625,000.

c.

$1,125,000.

d.

$1,875,000.


Valuation of Inventories: A Cost-Basis Approach

102.

8 - 67

a.

For 2012, cost of goods available for sale for Tate Corporation was $1,800,000. The gross profit rate was 20%. Sales for the year were $1,600,000. What was the amount of the ending inventory? $0.

b.

$520,000.

c.

$360,000.

d.

$320,000.

103.

On April 15 of the current year, a fire destroyed the entire uninsured inventory of a retail store. The following data are available: Sales, January 1 through April 15 Inventory, January 1 Purchases, January 1 through April 15 Markup on cost

$360,000 60,000 300,000 25%

a.

The amount of the inventory loss is estimated to be $72,000.

b.

$36,000.

c.

$90,000.

d.

$60,000.

104.

The inventory account of Irick Company at December 31, 2012, included the following items: Inventory Amount Merchandise out on consignment at sales price (including markup of 40% on selling price) $30,000 Goods purchased, in transit (shipped f.o.b. shipping point) 24,000 Goods held on consignment by Irick 26,000 Goods out on approval (sales price $15,200, cost $12,800) 15,200

a.

Based on the above information, the inventory account at December 31, 2012, should be reduced by $40,400.

b.

$45,200.


8 - 68

Test Bank for Intermediate Accounting, Fourteenth Edition

c.

$64,400.

d.

$64,000.

105.

The sales price for a product provides a gross profit of 20% of sales price. What is the gross profit as a percentage of cost? a. 20%. b. 17%. c. 25%. d. Not enough information is provided to determine.

106.

Gamma Ray Corp. has annual sales totaling $975,000 and an average gross profit of 20% of cost. What is the dollar amount of the gross profit? a. $195,000. b. $146,250. c. $162,500. d. $243,750.


Valuation of Inventories: A Cost-Basis Approach

8 - 69

107.

On August 31, a hurricane destroyed a retail location of Vinny's Clothier including the entire inventory on hand at the location. The inventory on hand as of June 30 totaled $640,000. Since June 30 until the time of the hurricane, the company made purchases of $170,000 and had sales of $500,000. Assuming the rate of gross profit to selling price is 40%, what is the approximate value of the inventory that was destroyed? a. $640,000. b. $363,000. c. $410,000. d. $510,000.

108.

On October 31, a fire destroyed PH Inc.'s entire retail inventory. The inventory on hand as of January 1 totaled $1,360,000. From January 1 through the time of the fire, the company made purchases of $330,000 and had sales of $720,000. Assuming the rate of gross profit to selling price is 40%, what is the approximate value of the inventory that was destroyed? a. $1,360,000. b. $1,346,000. c. $970,000. d. $1,258,000.

109.

On March 15, a fire destroyed Interlock Company's entire retail inventory. The inventory on hand as of January 1 totaled $3,300,000. From January 1 through the time of the fire, the company made purchases of $1,366,000, incurred freight-in of $156,000, and had sales of $2,420,000. Assuming the rate of gross profit to selling price is 30%, what is the approximate value of the inventory that was destroyed? a. $4,096,000. b. $2,972,000. c. $3,128,000. d. $4,822,000.

110.

Dicer uses the conventional retail method to determine its ending inventory at cost. Assume the beginning inventory at cost (retail) were $260,000 ($396,000), purchases during the current year at cost (retail) were $1,370,000 ($2,200,000), freight-in on these purchases totaled $86,000, sales during the current year totaled $2,100,000, and net markups (markdowns) were $48,000 ($72,000). What is the ending inventory value at cost? a. $306,328. b. $312,330. c. $314,824. d. $472,000.

111.

Boxer Inc. uses the conventional retail method to determine its ending inventory at cost. Assume the beginning inventory at cost (retail) were $196,500 ($297,000), purchases during the current year at cost (retail) were $1,704,000 ($2,596,800), freight-in on these purchases totaled $79,500, sales during the current year totaled $2,433,000, and net markups were $207,000. What is the ending inventory value at cost? a. $667,800. b. $523,098.


8 - 70

Test Bank for Intermediate Accounting, Fourteenth Edition

c. $426,723. d. $456,924.


Valuation of Inventories: A Cost-Basis Approach

8 - 71

112.

Barker Pet supply uses the conventional retail method to determine its ending inventory at cost. Assume the beginning inventory at cost (retail) were $531,200 ($653,800), purchases during the current year at cost (retail) were $2,137,200 ($2,772,200), freight-in on these purchases totaled $127,800, sales during the current year totaled $2,604,000, and net markups (markdowns) were $4,000 ($192,600). What is the ending inventory value at cost? a. $633,400. b. $516,222. c. $822,000. d. $493,334.

113.

Crane Sales Company uses the retail inventory method to value its merchandise inventory. The following information is available for the current year: Beginning inventory Purchases Freight-in Net markups Net markdowns Employee discounts Sales

Cost $ 30,000 175,000 2,500 — — — —

Retail $ 50,000 240,000 — 8,500 10,000 1,000 205,000

a.

If the ending inventory is to be valued at the lower-of-cost-or-market, what is the cost to retail ratio? $207,500 ÷ $290,000

b.

$207,500 ÷ $298,500

c.

$205,000 ÷ $300,000

d.

$207,500 ÷ $288,500

Use the following information for questions 114 through 118. The following data concerning the retail inventory method are taken from the financial records of Welch Company. Cost Retail Beginning inventory $ 98,000 $ 140,000 Purchases 448,000 640,000 Freight-in 12,000 — Net markups — 40,000 Net markdowns — 28,000 Sales — 672,000 114.

The ending inventory at retail should be


8 - 72

Test Bank for Intermediate Accounting, Fourteenth Edition

a.

$148,000.

b.

$120,000.

c.

$128,000.

d.

$84,000.


Valuation of Inventories: A Cost-Basis Approach

115.

8 - 73

a.

If the ending inventory is to be valued at approximately the lower of cost or market, the calculation of the cost to retail ratio should be based on goods available for sale at (1) cost and (2) retail, respectively of $558,000 and $820,000.

b.

$558,000 and $792,000.

c.

$558,000 and $780,000.

d.

$546,000 and $780,000.

116. a.

If the foregoing figures are verified and a count of the ending inventory reveals that merchandise actually on hand amounts to $108,000 at retail, the business has realized a windfall gain.

b.

sustained a loss.

c.

no gain or loss as there is close coincidence of the inventories.

d.

none of these.

*117. Assuming no change in the price level if the LIFO inventory method were used in conjunction with the data, the ending inventory at cost would be a. $85,200. b.

$84,000.

c.

$81,600.

d.

$86,400.

*118. Assuming that the LIFO inventory method were used in conjunction with the data and that the inventory at retail had increased during the period, then the computation of retail in the cost to retail ratio would a. exclude both markups and markdowns and include beginning inventory. b.

include markups and exclude both markdowns and beginning inventory.


8 - 74

Test Bank for Intermediate Accounting, Fourteenth Edition

c.

include both markups and markdowns and exclude beginning inventory.

d.

exclude markups and include both markdowns and beginning inventory.

119.

Drake Corporation had the following amounts, all at retail: Beginning inventory Purchase returns Abnormal shortage Sales Employee discounts

$ 3,600 6,000 4,000 72,000 1,600

a.

What is Drake’s ending inventory at retail? $74,400.

b.

$76,000.

c.

$77,600.

d.

$78,400

Purchases Net markups Net markdowns Sales returns Normal shortage

$140,000 18,000 2,800 1,800 2,600


Valuation of Inventories: A Cost-Basis Approach

120.

Goren Corporation had the following amounts, all at retail: Beginning inventory Purchase returns Abnormal shortage Sales Employee discounts

$ 3,600 6,000 4,000 72,000 1,600

Purchases Net markups Net markdowns Sales returns Normal shortage

a.

What is Goren’s ending inventory at retail? $44,400.

b.

$46,000.

c.

$47,600.

d.

$48,400

121.

Fry Corporation’s computation of cost of goods sold is: Beginning inventory Add: Cost of goods purchased Cost of goods available for sale Ending inventory Cost of goods sold

$ 60,000 530,000 590,000 90,000 $500,000

a.

The average days to sell inventory for Fry are 43.5 days.

b.

50.3 days.

c.

54.5 days.

d.

65.2 days.

122.

East Corporation’s computation of cost of goods sold is: Beginning inventory Add: Cost of goods purchased Cost of goods available for sale Ending inventory Cost of goods sold

a.

8 - 75

The average days to sell inventory for East are 68.3 days.

$ 60,000 482,000 542,000 80,000 $462,000

$110,000 18,000 2,800 1,800 2,600


8 - 76

Test Bank for Intermediate Accounting, Fourteenth Edition

b.

75.7 days.

c.

55.3 days.

d.

90.9 days.

123. a.

The 2012 financial statements of Sito Company reported a beginning inventory of $80,000, an ending inventory of $120,000, and cost of goods sold of $800,000 for the year. Sito’s inventory turnover ratio for 2012 is 10.0 times.

b.

8.0 times.

c.

6.7 times.

d.

5.7 times.


Valuation of Inventories: A Cost-Basis Approach

124.

8 - 77

Boxer Inc. reported inventory at the beginning of the current year of $360,000 and at the end of the current year of $411,000. If net sales for the current year are $3,321,900 and the corresponding cost of sales totaled $2,819,100, what is the inventory turnover ratio for the current year? a. 8.61. b. 6.86. c. 7.83. d. 7.31.

Use the following information for questions 125 through 129. Plank Co. uses the retail inventory method. The following information is available for the current year. Cost Retail Beginning inventory $ 156,000 $244,000 Purchases 590,000 830,000 Freight-in 10,000 — Employee discounts — 4,000 Net markups — 30,000 Net Markdowns — 40,000 Sales — 780,000 125. a.

If the ending inventory is to be valued at approximately lower of average cost or market, the calculation of the cost ratio should be based on cost and retail of $600,000 and $860,000.

b.

$600,000 and $856,000.

c.

$746,000 and $1,100,000.

d.

$756,000 and $1,104,000.

126. a.

The ending inventory at retail should be $320,000.

b.

$300,000.

c.

$288,000.

d.

$280,000.

127.

The approximate cost of the ending inventory by the conventional retail method is


8 - 78

Test Bank for Intermediate Accounting, Fourteenth Edition

a.

$191,800.

b.

$189,840.

c.

$196,000.

d.

$204,960.

*128. If the ending inventory is to be valued at approximately LIFO cost, the calculation of the cost ratio should be based on cost and retail of a. $756,000 and $1,104,000. b.

$756,000 and $1,064,000.

c.

$600,000 and $820,000.

d.

$600,000 and $860,000.


Valuation of Inventories: A Cost-Basis Approach

8 - 79

*129. Assuming that the LIFO inventory method is used, that the beginning inventory is the base inventory when the index was 100, and that the index at year end is 112, the ending inventory at dollar-value LIFO retail cost is a. $160,920. b.

$185,514.

c.

$191,800.

d.

$204,960.

Use the following information for questions 130 and 131. Eaton Company, which uses the retail LIFO method to determine inventory cost, has provided the following information for 2012: Cost Retail Inventory, 1/1/12 $ 141,000 $210,000 Net purchases 567,000 843,000 Net markups 102,000 Net markdowns 45,000 Net sales 795,000 *130. Assuming stable prices (no change in the price index during 2012), what is the cost of Eaton's inventory at December 31, 2012? a. $192,150. b.

$207,150.

c.

$204,000.

d.

$198,450.

*131. Assuming that the price index was 105 at December 31, 2012 and 100 at January 1, 2012, what is the cost of Eaton's inventory at December 31, 2012 under the dollar-value-LIFO retail method? a. $200,535. b.

$208,372.

c.

$210,458.


8 - 80

d.

Test Bank for Intermediate Accounting, Fourteenth Edition

$197,700.

Multiple Choice Answers—Computational


Valuation of Inventories: A Cost-Basis Approach

8 - 81

MULTIPLE CHOICE—CPA Adapted 132.

Ryan Distribution Co. has determined its December 31, 2012 inventory on a FIFO basis at $500,000. Information pertaining to that inventory follows: Estimated selling price Estimated cost of disposal Normal profit margin Current replacement cost

$510,000 20,000 60,000 450,000

a.

Ryan records losses that result from applying the lower-of-cost-or-market rule. At December 31, 2012, the loss that Ryan should recognize is $0.

b.

$10,000.

c.

$40,000.

d.

$50,000.

133. a.

Under the lower-of-cost-or-market method, the replacement cost of an inventory item would be used as the designated market value when it is below the net realizable value less the normal profit margin.

b.

when it is below the net realizable value and above the net realizable value less the normal

profit margin. c.

when it is above the net realizable value.

d.

regardless of net realizable value.

134.

a.

The original cost of an inventory item is above the replacement cost and the net realizable value. The replacement cost is below the net realizable value less the normal profit margin. As a result, under the lower-of-cost-or-market method, the inventory item should be reported at the net realizable value.

b.

net realizable value less the normal profit margin.

c.

replacement cost.


8 - 82

Test Bank for Intermediate Accounting, Fourteenth Edition

d.

original cost.

135.

Keen Company's accounting records indicated the following information: Inventory, 1/1/12 Purchases during 2012 Sales during 2012

$ 900,000 4,500,000 5,700,000

a.

A physical inventory taken on December 31, 2012, resulted in an ending inventory of $1,050,000. Keen's gross profit on sales has remained constant at 25% in recent years. Keen suspects some inventory may have been taken by a new employee. At December 31, 2012, what is the estimated cost of missing inventory? $75,000.

b.

$225,000.

c.

$300,000.

d.

$375,000.


Valuation of Inventories: A Cost-Basis Approach

136.

8 - 83

Henke Co. uses the retail inventory method to estimate its inventory for interim statement purposes. Data relating to the computation of the inventory at July 31, 2012, are as follows: Cost Retail Inventory, 2/1/12 $ 200,000 $ 250,000 Purchases 1,000,000 1,575,000 Markups, net 175,000 Sales 1,650,000 Estimated normal shoplifting losses 20,000 Markdowns, net 110,000

a.

Under the lower-of-cost-or-market method, Henke's estimated inventory at July 31, 2012 is $132,000.

b.

$144,000.

c.

$156,000.

d.

$220,000.

137.

At December 31, 2012, the following information was available from Kohl Co.'s accounting records: Cost Retail Inventory, 1/1/12 $147,000 $ 203,000 Purchases 833,000 1,155,000 Additional markups 42,000 Available for sale $980,000 $1,400,000

a.

Sales for the year totaled $1,150,000. Markdowns amounted to $10,000. Under the lowerof-cost-or-market method, Kohl's inventory at December 31, 2012 was $294,000.

b.

$175,000.

c.

$182,000.

d.

$168,000.

*138. On December 31, 2012, Pacer Co. adopted the dollar-value LIFO retail inventory method. Inventory data for 2013 are as follows: LIFO Cost Retail Inventory, 12/31/12 $450,000 $630,000 Inventory, 12/31/13 ? 825,000


8 - 84

Test Bank for Intermediate Accounting, Fourteenth Edition

Increase in price level for 2013 Cost to retail ratio for 2013

10% 70%

a.

Under the LIFO retail method, Pacer's inventory at December 31, 2013, should be $542,400.

b.

$577,500.

c.

$586,500.

d

$600,150.

Multiple Choice Answers—CPA Adapted


Valuation of Inventories: A Cost-Basis Approach

8 - 85

IFRS QUESTIONS True / False 1. IFRS permits an entity to reverse inventory write-downs in certain situations, whereas U.S. GAAP does not. 2. IFRS defines market as replacement cost subject to certain constraints. 3. IFRS uses a ceiling to determine market. 4. Similar to U.S. GAAP, certain agricultural products and mineral products can be reported at net realizable value using IFRS. 5. IFRS records market in the lower-of-cost-or-market differently than U.S. GAAP. Answers to True/False Multiple Choice Questions 1. Where is the authoritative IFRS guidance related to accounting and reporting for inventories found? a. IAS 2 b. IAS 18 c. IAS 41 d. All of these standards deal with inventory. 2. All of the following are key similarities between U.S. GAAP and IFRS with respect to accounting for inventories except a. guidelines on ownership of goods are similar. b. costs to include in inventories are similar. c. LIFO cost flow assumption where appropriate is used by both sets of standards. d. fair value valuation of inventories is prohibited by both sets of standards. 3. All of the following are key differences between U.S. GAAP and IFRS with respect to accounting for inventories except the a. definition of the lower-of-cost-or-market test for inventory valuation differs between U.S. GAAP and IFRS. b. inventory basis determination for writedowns differs between U.S. GAAP and IFRS. c. guidelines are more principles based under IFRS than they are under U.S. GAAP. d. average costing method is prohibited under IFRS.


8 - 86

Test Bank for Intermediate Accounting, Fourteenth Edition

4. Alonzo Company in Italy prepares its financial statements in accordance with IFRS. In 2012, it reported cost of goods sold of â‚Ź600 million and average inventory of â‚Ź150 million. What is Alonzo's inventory turnover ratio? a. 4 days b. 25 days c. 91.25 days d. 100 days 5. Starfish Company (a company using U.S. GAAP and LIFO inventory method) is considering changing to IFRS and the FIFO inventory method. How would a comparison of these methods affect Starfish's financials? a. During a period of inflation, the current ratio would decrease when IFRS and the FIFO inventory method are used as compared to U.S. GAAP and LIFO. b. During a period of inflation, the taxes will decrease when IFRS and the FIFO inventory method are used as compared to U.S. GAAP and LIFO. c. During a period of inflation, net income would be greater if IFRS and the FIFO inventory method are used as compared to U.S.GAAP and LIFO. d. During a period of inflation, working capital would decrease when IFRS and the FIFO inventory method are used as compared to U.S. GAAP and LIFO. 6. Which of the following statements is true regarding IFRS and inventories? a. In order to determine market valuation of inventories, IFRS uses a ceiling and a floor. b. IFRS permits the option of valuing inventories at fair value. c. With respect to inventories, IFRS defines market as net realizable value. d. IFRS allows inventory to be written up above its original cost. 7. State Company manufactured a forklift machine at a cost of $60,000. The product is sold for $66,000 at a 5% discount. The delivery costs are estimated to be $6,000. Under IFRS, how much should be the carrying amount of this inventory? a. $60,000 b. $66,000 c. $54,000 d. $56,700 8. The following information relates to Moore Company's inventory: Cost of inventory = $860 Selling price of inventory = $1,000 Normal profit margin = 10% of selling price Current replacement cost = $740 Cost of completion and disposal = $100 Under IFRS, which of the following would be the correct measurement value for the inventory? a. $860 b. $740 c. $1,000 d. $900


Valuation of Inventories: A Cost-Basis Approach

8 - 87

9. Assume that Darcy Industries had the following inventory values: Inventory cost (on December 31, 2011) = $1,500 Inventory market (on December 31, 2011) = $1,350 Inventory net realizable value (on December 31, 2011) = $1,320 Inventory market (on June 30, 2012) = $1,560 Inventory net realizable value (on June 30, 2012) = $1,570 Under IFRS, what is the inventory carrying value on December 31, 2011? a. $1,500 b. $1,350 c. $1,320 d. $1,390 10. Assume that Darcy Industries had the following inventory values: Inventory cost (on December 31, 2011) = $1,500 Inventory market (on December 31, 2011) = $1,350 Inventory net realizable value (on December 31, 2011) = $1,320 Inventory market (on June 30, 2012) = $1,560 Inventory net realizable value (on June 30, 2012) = $1,570 Under IFRS, what is the inventory carrying value on June 30, 2012? a. $1,500 b. $1,560 c. $1,570 d. $1,320 Answers to Multiple Choice Short Answer 1. Briefly describe some of the similarities and differences between U.S. GAAP and IFRS with respect to the accounting for inventories.

2. Explain the main obstacle to achieving convergence in the area of inventory accounting.

CHAPTER 10 ACQUISITION AND DISPOSITION OF PROPERTY, PLANT, AND EQUIPMENT IFRS questions are available at the end of this chapter.


8 - 88

Test Bank for Intermediate Accounting, Fourteenth Edition

TRUE-FALSE—Conceptual 1. Assets classified as Property, Plant, and Equipment can be either acquired for use in operations, or acquired for resale. 2. Assets classified as Property, Plant, and Equipment must be both long-term in nature and possess physical substance. 3. When land with an old building is purchased as a future building site, the cost of removing the old building is part of the cost of the new building. 4. Insurance on equipment purchased, while the equipment is in transit, is part of the cost of the equipment. 5. Special assessments for local improvements such as street lights and sewers should be accounted for as land improvements. 6. Variable overhead costs incurred to self-construct an asset should be included in the cost of the asset. 7. Companies should assign no portion of fixed overhead to self-constructed assets. 8. When capitalizing interest during construction of an asset, an imputed interest cost on stock financing must be included. 9. Assets under construction for a company’s own use do not qualify for interest cost capitalization. 10. Avoidable interest is the amount of interest cost that a company could theoretically avoid if it had not made expenditures for the asset. 11. When a company purchases land with the intention of developing it for a particular use, interest costs associated with those expenditures qualify for interest capitalization. 12. Assets purchased on long-term credit contracts should be recorded at the present value of the consideration exchanged. 13. Companies account for the exchange of nonmonetary assets on the basis of the fair value of the asset given up or the fair value of the asset received. 14. If a nonmonetary exchange lacks commercial substance, and cash is received, a partial gain or loss is recognized. 15. When a company exchanges nonmonetary assets and a loss results, the company recognizes the loss only if the exchange has commercial substance.


Valuation of Inventories: A Cost-Basis Approach

8 - 89

16. Costs incurred subsequent to the acquisition of an asset are capitalized if they provide future benefits. 17. Improvements are often referred to as betterments and involve the substitution of a better asset for the one currently used. 18. When an ordinary repair occurs, several periods will usually benefit. 19. Companies always treat gains or losses from an involuntary conversion as extraordinary items. 20. If a company scraps an asset without any cash recovery, it recognizes a loss equal to the asset’s book value. True False Answers—Conceptual

MULTIPLE CHOICE—Conceptual 21.

Plant assets may properly include a. deposits on machinery not yet received. b. idle equipment awaiting sale. c. land held for possible use as a future plant site. d. none of these.

22.

Which of the following is not a major characteristic of a plant asset? a. Possesses physical substance b. Acquired for resale c. Acquired for use d. Yields services over a number of years

23.

Which of these is not a major characteristic of a plant asset? a. Possesses physical substance b. Acquired for use in operations c. Yields services over a number of years d. All of these are major characteristics of a plant asset.

24.

Cotton Hotel Corporation recently purchased Emporia Hotel and the land on which it is located with the plan to tear down the Emporia Hotel and build a new luxury hotel on the site. The cost of the Emporia Hotel should be a. depreciated over the period from acquisition to the date the hotel is scheduled to be torn down. b. written off as an extraordinary loss in the year the hotel is torn down. c. capitalized as part of the cost of the land. d. capitalized as part of the cost of the new hotel.


8 - 90

Test Bank for Intermediate Accounting, Fourteenth Edition


Valuation of Inventories: A Cost-Basis Approach

8 - 91

25.

The cost of land does not include a. costs of grading, filling, draining, and clearing. b. costs of removing old buildings. c. costs of improvements with limited lives. d. special assessments.

26.

The cost of land typically includes the purchase price and all of the following costs except a. grading, filling, draining, and clearing costs. b. street lights, sewers, and drainage systems cost. c. private driveways and parking lots. d. assumption of any liens or mortgages on the property.

27.

If a corporation purchases a lot and building and subsequently tears down the building and uses the property as a parking lot, the proper accounting treatment of the cost of the building would depend on a. the significance of the cost allocated to the building in relation to the combined cost of the lot and building. b. the length of time for which the building was held prior to its demolition. c. the contemplated future use of the parking lot. d. the intention of management for the property when the building was acquired.

28.

The debit for a sales tax properly levied and paid on the purchase of machinery preferably would be a charge to a. the machinery account. b. a separate deferred charge account. c. miscellaneous tax expense (which includes all taxes other than those on income). d. accumulated depreciation--machinery.

29.

Fences and parking lots are reported on the balance sheet as a. current assets. b. land improvements. c. land. d. property and equipment.

S

Historical cost is the basis advocated for recording the acquisition of property, plant, and equipment for all of the following reasons except a. at the date of acquisition, cost reflects fair market value. b. property, plant, and equipment items are always acquired at their original historical cost. c. historical cost involves actual transactions and, as such, is the most reliable basis. d. gains and losses should not be anticipated but should be recognized when the asset is sold.

S

To be consistent with the historical cost principle, overhead costs incurred by an enterprise constructing its own building should be a. allocated on the basis of lost production. b. eliminated completely from the cost of the asset.

30.

31.


8 - 92

Test Bank for Intermediate Accounting, Fourteenth Edition

c. allocated on an opportunity cost basis. d. allocated on a pro rata basis between the asset and normal operations.


Valuation of Inventories: A Cost-Basis Approach

8 - 93

32.

Which of the following costs are capitalized for self-constructed assets? a. Materials and labor only b. Labor and overhead only c. Materials and overhead only d. Materials, labor, and overhead

33.

Which of the following assets do not qualify for capitalization of interest costs incurred during construction of the assets? a. Assets under construction for an enterprise's own use. b. Assets intended for sale or lease that are produced as discrete projects. c. Assets financed through the issuance of long-term debt. d. Assets not currently undergoing the activities necessary to prepare them for their intended use.

34.

Assets that qualify for interest cost capitalization include a. assets under construction for a company's own use. b. assets that are ready for their intended use in the earnings of the company. c. assets that are not currently being used because of excess capacity. d. All of these assets qualify for interest cost capitalization.

35.

When computing the amount of interest cost to be capitalized, the concept of "avoidable interest" refers to a. the total interest cost actually incurred. b. a cost of capital charge for stockholders' equity. c. that portion of total interest cost which would not have been incurred if expenditures for asset construction had not been made. d. that portion of average accumulated expenditures on which no interest cost was incurred.

36.

The period of time during which interest must be capitalized ends when a. the asset is substantially complete and ready for its intended use. b. no further interest cost is being incurred. c. the asset is abandoned, sold, or fully depreciated. d. the activities that are necessary to get the asset ready for its intended use have begun.

37.

Which of the following statements is true regarding capitalization of interest? a. Interest cost capitalized in connection with the purchase of land to be used as a building site should be debited to the land account and not to the building account. b. The amount of interest cost capitalized during the period should not exceed the actual interest cost incurred. c. When excess borrowed funds not immediately needed for construction are temporarily invested, any interest earned should be offset against interest cost incurred when determining the amount of interest cost to be capitalized. d. The minimum amount of interest to be capitalized is determined by multiplying a weighted average interest rate by the amount of average accumulated expenditures on qualifying assets during the period.


8 - 94

Test Bank for Intermediate Accounting, Fourteenth Edition

38.

Construction of a qualifying asset is started on April 1 and finished on December 1. The fraction used to multiply an expenditure made on April 1 to find weighted-average accumulated expenditures is a. 8/8. b. 8/12. c. 9/12. d. 11/12.

39.

When funds are borrowed to pay for construction of assets that qualify for capitalization of interest, the excess funds not needed to pay for construction may be temporarily invested in interest-bearing securities. Interest earned on these temporary investments should be a. offset against interest cost incurred during construction. b. used to reduce the cost of assets being constructed. c. multiplied by an appropriate interest rate to determine the amount of interest to be capitalized. d. recognized as revenue of the period.

40.

Interest cost that is capitalized should a. be written off over the remaining term of the debt. b. be accumulated in a separate deferred charge account and written off equally over a 40year period. c. not be written off until the related asset is fully depreciated or disposed of. d. none of these.

S

Which of the following is not a condition that must be satisfied before interest capitalization can begin on a qualifying asset? a. Interest cost is being incurred. b. Expenditures for the assets have been made. c. The interest rate is equal to or greater than the company's cost of capital. d. Activities that are necessary to get the asset ready for its intended use are in progress.

S

Which of the following is the recommended approach to handling interest incurred in financing the construction of property, plant and equipment? a. Capitalize only the actual interest costs incurred during construction. b. Charge construction with all costs of funds employed, whether identifiable or not. c. Capitalize no interest during construction. d. Capitalize interest costs equal to the prime interest rate times the estimated cost of the asset being constructed.

S

Which of the following nonmonetary exchange transactions represents a culmination of the earning process? a. Exchange of assets with no difference in future cash flows. b. Exchange of products by companies in the same line of business with no difference in future cash flows. c. Exchange of assets with a difference in future cash flows. d. Exchange of an equivalent interest in similar productive assets that causes the companies involved to remain in essentially the same economic position.

41.

42.

43.


Valuation of Inventories: A Cost-Basis Approach

8 - 95


8 - 96

Test Bank for Intermediate Accounting, Fourteenth Edition

S

When boot is involved in an exchange having commercial substance. a. gains or losses are recognized in their entirely. b. a gain or loss is computed by comparing the fair value of the asset received with the fair value of the asset given up. c. only gains should be recognized. d. only losses should be recognized.

S

The cost of a nonmonetary asset acquired in exchange for another nonmonetary asset and the exchange has commercial substance is usually recorded at a. the fair value of the asset given up, and a gain or loss is recognized. b. the fair value of the asset given up, and a gain but not a loss may be recognized. c. the fair value of the asset received if it is equally reliable as the fair value of the asset given up. d. either the fair value of the asset given up or the asset received, whichever one results in the largest gain (smallest loss) to the company.

P

46.

Ringler Corporation exchanges one plant asset for a similar plant asset and gives cash in the exchange. The exchange is not expected to cause a material change in the future cash flows for either entity. If a gain on the disposal of the old asset is indicated, the gain will a. be reported in the Other Revenues and Gains section of the income statement. b. effectively reduce the amount to be recorded as the cost of the new asset. c. effectively increase the amount to be recorded as the cost of the new asset. d. be credited directly to the owner's capital account.

47.

Plant assets purchased on long-term credit contracts should be accounted for at a. the total value of the future payments. b. the future amount of the future payments. c. the present value of the future payments. d. none of these.

48.

When a plant asset is acquired by issuance of common stock, the cost of the plant asset is properly measured by the a. par value of the stock. b. stated value of the stock. c. book value of the stock. d. fair value of the stock.

49.

When a closely held corporation issues preferred stock for land, the land should be recorded at the a. total par value of the stock issued. b. total book value of the stock issued. c. total liquidating value of the stock issued. d. fair value of the land.

50.

Accounting recognition should be given to some or all of the gain realized on a nonmonetary exchange of plant assets except when the exchange has a. no commercial substance and additional cash is paid.

44.

45.


Valuation of Inventories: A Cost-Basis Approach

8 - 97

b. no commercial substance and additional cash is received. c. commercial substance and additional cash is paid. d. commercial substance and additional cash is received. 51.

For a nonmonetary exchange of plant assets, accounting recognition should not be given to a. a loss when the exchange has no commercial substance. b. a gain when the exchange has commercial substance. c. part of a gain when the exchange has no commercial substance and cash is paid (cash paid/received is less than 25% of the fair value of the exchange). d. part of a gain when the exchange has no commercial substance and cash is received (cash paid or received is less than 25% of the fair value of the exchange).

52.

When an enterprise is the recipient of a donated asset, the account credited may be a a. paid-in capital account. b. revenue account. c. deferred revenue account. d. all of these.

53.

A plant site donated by a township to a manufacturer that plans to open a new factory should be recorded on the manufacturer's books at a. the nominal cost of taking title to it. b. its fair value. c. one dollar (since the site cost nothing but should be included in the balance sheet). d. the value assigned to it by the company's directors.

54.

In order for a cost to be capitalized (capital expenditure), the following must be present: a. The useful life of an asset must be increased. b. The quantity of assets must be increased. c. The quality of assets must be increased. d. Any one of these.

55.

An improvement made to a machine increased its fair value and its production capacity by 25% without extending the machine's useful life. The cost of the improvement should be a. expensed. b. debited to accumulated depreciation. c. capitalized in the machine account. d. allocated between accumulated depreciation and the machine account.

56.

Which of the following is a capital expenditure? a. Payment of an account payable b. Retirement of bonds payable c. Payment of Federal income taxes d. None of these

57.

Which of the following is not a capital expenditure? a. Repairs that maintain an asset in operating condition b. An addition


8 - 98

Test Bank for Intermediate Accounting, Fourteenth Edition

c. A betterment d. A replacement


Valuation of Inventories: A Cost-Basis Approach

8 - 99

P

In accounting for plant assets, which of the following outlays made subsequent to acquisition should be fully expensed in the period the expenditure is made? a. Expenditure made to increase the efficiency or effectiveness of an existing asset b. Expenditure made to extend the useful life of an existing asset beyond the time frame originally anticipated c. Expenditure made to maintain an existing asset so that it can function in the manner intended d. Expenditure made to add new asset services

S

An expenditure made in connection with a machine being used by an enterprise should be a. expensed immediately if it merely extends the useful life but does not improve the quality. b. expensed immediately if it merely improves the quality but does not extend the useful life. c. capitalized if it maintains the machine in normal operating condition. d. capitalized if it increases the quantity of units produced by the machine.

S

60.

When a plant asset is disposed of, a gain or loss may result. The gain or loss would be classified as an extraordinary item on the income statement if it resulted from a. an involuntary conversion and the conditions of the disposition are unusual and infrequent in nature. b. a sale prior to the completion of the estimated useful life of the asset. c. the sale of a fully depreciated asset. d. an abandonment of the asset.

61.

The sale of a depreciable asset resulting in a loss indicates that the proceeds from the sale were a. less than current fair value. b. greater than cost. c. greater than book value. d. less than book value.

62.

Which of the following statements about involuntary conversions is false? a. An involuntary conversion may result from condemnation or fire. b. The gain or loss from an involuntary conversion may be reported as an extraordinary item. c. The gain or loss from an involuntary conversion should not be recognized when the enterprise reinvests in replacement assets. d. All of these.

58.

59.


8 - 100 Test Bank for Intermediate Accounting, Fourteenth Edition

Multiple Choice Answers—Conceptual Solutions to those Multiple Choice questions for which the answer is “none of these.” 21.

Long-lived tangible assets used in the enterprise’s operations.

40.

Capitalized interest is depreciated over the related asset’s useful life.

56.

Capital expenditures include additions, betterments, improvements, and extraordinary repairs.

MULTIPLE CHOICE—Computational Use the following information for questions 63 and 64. Wilson Co. purchased land as a factory site for $800,000. Wilson paid $80,000 to tear down two buildings on the land. Salvage was sold for $5,400. Legal fees of $3,480 were paid for title investigation and making the purchase. Architect's fees were $31,200. Title insurance cost $2,400, and liability insurance during construction cost $2,600. Excavation cost $10,440. The contractor was paid $2,500,000. An assessment made by the city for pavement was $6,400. Interest costs during construction were $170,000. 63.

The cost of the land that should be recorded by Wilson Co. is a. $880,480. b. $886,880. c. $889,880. d. $896,280.

64.

The cost of the building that should be recorded by Wilson Co. is a. $2,503,800. b. $2,504,840. c. $2,513,200. d. $2,514,240.


Valuation of Inventories: A Cost-Basis Approach

65.

8 - 101

On February 1, 2012, Nelson Corporation purchased a parcel of land as a factory site for $250,000. An old building on the property was demolished, and construction began on a new building which was completed on November 1, 2012. Costs incurred during this period are listed below: Demolition of old building $ 20,000 Architect's fees 35,000 Legal fees for title investigation and purchase contract 5,000 Construction costs 1,290,000 (Salvaged materials resulting from demolition were sold for $10,000.) Nelson should record the cost of the land and new building, respectively, as a. $275,000 and $1,315,000. b. $260,000 and $1,330,000. c. $260,000 and $1,325,000. d. $265,000 and $1,325,000.

66.

Worthington Chandler Company purchased equipment for $12,000. Sales tax on the purchase was $800. Other costs incurred were freight charges of $200, repairs of $350 for damage during installation, and installation costs of $225. What is the cost of the equipment? a. $12,000 b. $12,800 c. $13,225 d. $13,575

67.

Fogelberg Company purchased equipment for $15,000. Sales tax on the purchase was $900. Other costs incurred were freight charges of $240, repairs of $420 for damage during installation, and installation costs of $270. What is the cost of the equipment? a. $15,000. b. $15,900. c. $16,410. d. $16,830.

68.

During self-construction of an asset by Samuelson Company, the following were among the costs incurred: Fixed overhead for the year Portion of $1,000,000 fixed overhead that would be allocated to asset if it were normal production Variable overhead attributable to self-construction

$1,000,000 50,000 35,000

What amount of overhead should be included in the cost of the self-constructed asset? a. $ -0b. $35,000 c. $50,000 d. $85,000


8 - 102 Test Bank for Intermediate Accounting, Fourteenth Edition

69.

During self-construction of an asset by Richardson Company, the following were among the costs incurred: Fixed overhead for the year Portion of $1,000,000 fixed overhead that would be allocated to asset if it were normal production Variable overhead attributable to self-construction

$1,000,000 60,000 75,000

What amount of overhead should be included in the cost of the self-constructed asset? a. $ -0b. $ 60,000 c. $ 75,000 d. $135,000 70.

Mendenhall Corporation constructed a building at a cost of $10,000,000. Average accumulated expenditures were $4,000,000, actual interest was $600,000, and avoidable interest was $400,000. If the salvage value is $800,000, and the useful life is 40 years, depreciation expense for the first full year using the straight-line method is a. $240,000. b. $245,000. c. $260,000. d. $340,000.

71.

Messersmith Company is constructing a building. Construction began in 2012 and the building was completed 12/31/12. Messersmith made payments to the construction company of $1,500,000 on 7/1, $3,150,000 on 9/1, and $3,000,000 on 12/31. Average accumulated expenditures were a. $1,537,500. b. $1,800,000. c. $4,650,000. d. $7,650,000.

72.

Huffman Corporation constructed a building at a cost of $20,000,000. Average accumulated expenditures were $8,000,000, actual interest was $1,200,000, and avoidable interest was $800,000. If the salvage value is $1,600,000, and the useful life is 40 years, depreciation expense for the first full year using the straight-line method is a. $480,000. b. $490,000. c. $520,000. d. $680,000.

73.

Gutierrez Company is constructing a building. Construction began in 2012 and the building was completed 12/31/12. Gutierrez made payments to the construction company of $2,000,000 on 7/1, $4,400,000 on 9/1, and $4,000,000 on 12/31. Average accumulated expenditures were a. $2,100,000. b. $2,466,667. c. $6,400,000.


Valuation of Inventories: A Cost-Basis Approach

d. $10,400,000.

8 - 103


8 - 104 Test Bank for Intermediate Accounting, Fourteenth Edition

74.

On May 1, 2012, Goodman Company began construction of a building. Expenditures of $240,000 were incurred monthly for 5 months beginning on May 1. The building was completed and ready for occupancy on September 1, 2012. For the purpose of determining the amount of interest cost to be capitalized, the average accumulated expenditures on the building during 2012 were a. $200,000. b. $240,000. c. $960,000. d. $1,200,000.

75.

During 2012, Kimmel Co. incurred average accumulated expenditures of $600,000 during construction of assets that qualified for capitalization of interest. The only debt outstanding during 2012 was a $750,000, 10%, 5-year note payable dated January 1, 2010. What is the amount of interest that should be capitalized by Kimmel during 2012? a. $0. b. $15,000. c. $60,000. d. $75,000.

76.

On March 1, Felt Co. began construction of a small building. Payments of $160,000 were made monthly for three months beginning March 1. The building was completed and ready for occupancy on June 1. In determining the amount of interest cost to be capitalized, the weighted-average accumulated expenditures are a. $40,000. b. $80,000. c. $160,000. d. $320,000.

77.

On March 1, Imhoff Co. began construction of a small building. Payments of $240,000 were made monthly for four months beginning March 1. The building was completed and ready for occupancy on June 1. In determining the amount of interest cost to be capitalized, the weighted-average accumulated expenditures are a. $120,000. b. $240,000. c. $480,000. d. $960,000.

Use the following information for questions 78 through 80. On March 1, 2012, Newton Company purchased land for an office site by paying $900,000 cash. Newton began construction on the office building on March 1. The following expenditures were incurred for construction: Date Expenditures March 1, 2012 $ 600,000 April 1, 2012 840,000 May 1, 2012 1,500,000 June 1, 2012 2,400,000


Valuation of Inventories: A Cost-Basis Approach

8 - 105


8 - 106 Test Bank for Intermediate Accounting, Fourteenth Edition

The office was completed and ready for occupancy on July 1. To help pay for construction, $1,200,000 was borrowed on March 1, 2012 on a 9%, 3-year note payable. Other than the construction note, the only debt outstanding during 2012 was a $500,000, 12%, 6-year note payable dated January 1, 2012. 78.

The weighted-average accumulated expenditures on the construction project during 2012 were a. $640,000. b. $4,890,000. c. $520,000. d. $1,160,000.

79.

The actual interest cost incurred during 2012 was a. $150,000. b. $168,000. c. $84,000. d. $140,000.

80.

Assume the weighted-average accumulated expenditures for the construction project are $870,000. The amount of interest cost to be capitalized during 2012 is a. $130,500. b. $138,000. c. $150,000. d. $168,000.

81.

During 2012, Bass Corporation constructed assets costing $2,000,000. The weightedaverage accumulated expenditures on these assets during 2012 was $600,000. To help pay for construction, $880,000 was borrowed at 10% on January 1, 2012, and funds not needed for construction were temporarily invested in short-term securities, yielding $18,000 in interest revenue. Other than the construction funds borrowed, the only other debt outstanding during the year was a $1,000,000, 10-year, 9% note payable dated January 1, 2006. What is the amount of interest that should be capitalized by Bass during 2012? a. $120,000. b. $60,000. c. $116,800. d. $188,800.

Use the following information for questions 82 through 85. On January 2, 2012, Indian River Groves began construction of a new citrus processing plant. The automated plant was finished and ready for use on September 30, 2013. Expenditures for the construction were as follows: January 2, 2012 September 1, 2012 December 31, 2012 March 31, 2013

$300,000 900,000 900,000 900,000


Valuation of Inventories: A Cost-Basis Approach

September 30, 2013

8 - 107

600,000

Indian River Groves borrowed $1,650,000 on a construction loan at 12% interest on January 2, 2012. This loan was outstanding during the construction period. The company also had $6,000,000 in 9% bonds outstanding in 2012 and 2013. 82.

What were the weighted-average accumulated expenditures for 2012? a. $800,000 b. $750,000 c. $600,000 d. $1,500,000

83.

The interest capitalized for 2012 was: a. $270,000 b. $72,000 c. $228,000 d. $90,000

84.

What were the weighted-average accumulated expenditures for 2013 by the end of the construction period? a. $585,000 b. $2,452,500 c. $2,979,000 d. $2,079,000

85.

The interest capitalized for 2013 was: a. $187,110 b. $177,458 c. $ 38,610 d. $ 148,500

Use the following information to answer questions 86 - 90. Arlington Company is constructing a building. Construction began on January 1 and was completed on December 31. Expenditures were $4,000,000 on March 1, $3,300,000 on June 1, and $5,000,000 on December 31. Arlington Company borrowed $2,000,000 on January 1 on a 5year, 12% note to help finance construction of the building. In addition, the company had outstanding all year a 10%, 3-year, $4,000,000 note payable and an 11%, 4-year, $7,500,000 note payable. 86.

What are the weighted-average accumulated expenditures? a. $7,300,000 b. $5,258,333 c. $12,300,000 d. $6,150,000

87.

What is the weighted-average interest rate used for interest capitalization purposes?


8 - 108 Test Bank for Intermediate Accounting, Fourteenth Edition

a. b. c. d.

11% 10.85% 10.5% 10.65%


Valuation of Inventories: A Cost-Basis Approach

8 - 109

88.

What is the avoidable interest for Arlington Company? a. $240,000 b. $773,013 c. $273,802 d. $587,012

89.

What is the actual interest for Arlington Company? a. $1,465,000 b. $1,485,000 c. $1,225,000 d. $587,012

90.

What amount of interest should be charged to expense? a. $637,987 b. $1,225 c. $877,987 d. $691,987

91.

Dodson Company traded in a manual pressing machine for an automated pressing machine and gave $16,000 cash. The old machine cost $186,000 and had a net book value of $142,000. The old machine had a fair value of $120,000. Which of the following is the correct journal entry to record the exchange? a. Equipment 136,000 Loss on Disposal 22,000 Accumulated Depreciation 44,000 Equipment 186,000 Cash 16,000 b. Equipment 136,000 Equipment 120,000 Cash 16,000 c. Cash 16,000 Equipment 120,000 Loss on Disposal 22,000 Accumulated Depreciation 44,000 Equipment 202,000 d. Equipment 246,000 Accumulated Depreciation 44,000 Equipment 186,000 Cash 16,000

Use the following information to answer questions 92 & 93. Below is the information relative to an exchange of assets by Stanton Company. The exchange lacks commercial substance. Old Equipment


8 - 110

Test Bank for Intermediate Accounting, Fourteenth Edition

Case I Case II 92.

Fair Value $225,000 $135,000

Cash Paid $45,000 $21,000

Which of the following would be correct for Stanton to record in Case I? a. b. c. d.

93.

Book Value $225,000 $150,000

Record Equipment at: $270,000 $300,000 $225,000 $270,000

Record a gain of (loss) of: $0 $30,000 $(15,000) $30,000

Which of the following would be correct for Stanton to record in Case II? a. b. c. d.

Record Equipment at: $171,000 $150,000 $156,000 $150,000

Record a gain of (loss) of: $15,000 $6,000 $(15,000) $(6,000)

Use the following information for questions 94 and 95. Glen Inc. and Armstrong Co. have an exchange with no commercial substance. The asset given up by Glen Inc. has a book value of $48,000 and a fair value of $60,000. The asset given up by Armstrong Co. has a book value of $80,000 and a fair value of $76,000. Boot of $16,000 is received by Armstrong Co. 94. What amount should Glen Inc. record for the asset received? a. $60,000 b. $64,000 c. $76,000 d. $80,000 95.

What amount should Armstrong Co. record for the asset received? a. $60,000 b. $64,000 c. $76,000 d. $80,000

96.

Hardin Company received $60,000 in cash and a used computer with a fair value of $180,000 from Page Corporation for Hardin Company's existing computer having a fair value of $240,000 and an undepreciated cost of $225,000 recorded on its books. The transaction has no commercial substance. How much gain should Hardin recognize on this exchange, and at what amount should the acquired computer be recorded, respectively? a. $0 and $165,000 b. $1,153 and $166,153 c. $15,000 and $180,000 d. $60,000 and $225,000


Valuation of Inventories: A Cost-Basis Approach

8 - 111

Use the following information to answer questions 97 & 98. Jamison Company purchased the assets of Booker Company at an auction for $2,800,000. An independent appraisal of the fair value of the assets is listed below: Land $950,000 Building 1,400,000 Equipment 1,050,000 Trucks 1,700,000


8 - 112

Test Bank for Intermediate Accounting, Fourteenth Edition

97.

Assuming that specific identification costs are impracticable and that Jamison allocates the purchase price on the basis of the relative fair values, what amount would be allocated to the Trucks? a. $933,333 b. $1,400,000 c. $1,680,000 d. $1,700,000

98.

Assuming that specific identification costs are impracticable and that Jamison allocates the purchase price on the basis of the relative fair values, what amount would be allocated to the Building? a. $1,059,460 b. $1,400,000 c. $2,550,000 d. $768,627

99.

On December 1, Miser Corporation exchanged 3,000 shares of its $25 par value common stock held in treasury for a parcel of land to be held for a future plant site. The treasury shares were acquired by Miser at a cost of $40 per share, and on the exchange date the common shares of Miser had a fair value of $50 per share. Miser received $9,000 for selling scrap when an existing building on the property was removed from the site. Based on these facts, the land should be capitalized at a. $111,000. b. $120,000. c. $141,000. d. $150,000.

100.

Storm Corporation purchased a new machine on October 31, 2012. A $3,600 down payment was made and three monthly installments of $10,800 each are to be made beginning on November 30, 2012. The cash price would have been $34,800. Storm paid no installation charges under the monthly payment plan but a $600 installation charge would have been incurred with a cash purchase. The amount to be capitalized as the cost of the machine on October 31, 2012 would be a. $36,600. b. $36,000. c. $35,400. d. $34,800.

101.

Horner Company buys a delivery van with a list price of $45,000. The dealer grants a 15% reduction in list price and an additional 2% cash discount on the net price if payment is made in 30 days. Sales taxes amount to $600 and the company paid an extra $450 to have a special device installed. What should be the recorded cost of the van? a. $37,485. b. $38,468. c. $38,535. d. $38,085.


Valuation of Inventories: A Cost-Basis Approach

8 - 113

102.

On August 1, 2012, Hayes Corporation purchased a new machine on a deferred payment basis. A down payment of $9,000 was made and 4 monthly installments of $7,500 each are to be made beginning on September 1, 2012. The cash equivalent price of the machine was $36,000. Hayes incurred and paid installation costs amounting to $1,500. The amount to be capitalized as the cost of the machine is a. $36,000. b. $37,500. c. $39,000. d. $40,500.

103.

On April 1, Mooney Corporation purchased for $1,710,000 a tract of land on which was located a warehouse and office building. The following data were collected concerning the property: Current Assessed ValuationVendor’s Original Cost Land $600,000 $560,000 Warehouse 400,000 360,000 Office building 800,000 680,000 $1,800,000 $1,600,000 What are the appropriate amounts that Mooney should record for the land, warehouse, and office building, respectively? a. Land, $560,000; warehouse, $360,000; office building, $680,000. b. Land, $600,000; warehouse, $400,000; office building, $800,000. c. Land, $598,500; warehouse, $384,750; office building, $363,375. d. Land, $570,000; warehouse, $380,000; office building, $760,000.

104.

On August 1, 2012, Mendez Corporation purchased a new machine on a deferred payment basis. A down payment of $2,000 was made and 4 annual installments of $18,000 each are to be made beginning on September 1, 2012. The cash equivalent price of the machine was $69,000. Due to an employee strike, Mendez could not install the machine immediately, and thus incurred $900 of storage costs. Costs of installation (excluding the storage costs) amounted to $2,400. The amount to be capitalized as the cost of the machine is a. $69,000. b. $71,400. c. $72,300. d. $78,000.


8 - 114

Test Bank for Intermediate Accounting, Fourteenth Edition

105.

Siegle Company exchanged 600 shares of Guinn Company common stock, which Siegle was holding as an investment, for equipment from Mayo Company. The Guinn Company common stock, which had been purchased by Siegle for $50 per share, had a quoted market value of $58 per share at the date of exchange. The equipment had a recorded amount on Mayo's books of $31,500. What journal entry should Siegle make to record this exchange? a. Equipment ............................................................................. 30,000 Investment in Guinn Co. Common Stock .................... 30,000 b. Equipment ............................................................................. 31,500 Investment in Guinn Co. Common Stock .................... 30,000 Gain on Disposal of Investment ................................... 1,500 c. Equipment ............................................................................. 31,500 Loss on Disposal of Investment ............................................ 3,300 Investment in Guinn Co. Common Stock .................... 34,800 d. Equipment ............................................................................. 34,800 Investment in Guinn Co. Common Stock .................... 30,000 Gain on Disposal of Investment ................................... 4,800

106.

On January 2, 2012, Rapid Delivery Company traded in an old delivery truck for a newer model. The exchange lacked commercial substance. Data relative to the old and new trucks follow: Old Truck Original cost $36,000 Accumulated depreciation as of January 2, 2012 24,000 Average published retail value 11,000 New Truck List price $60,000 Cash price without trade-in 54,000 Cash paid with trade-in 45,000 What should be the cost of the new truck for financial accounting purposes? a. $45,000. b. $54,000. c. $57,000. d. $60,000.

107.

On December 1, 2012, Kelso Company acquired new equipment in exchange for old equipment that it had acquired in 2009. The old equipment was purchased for $70,000 and had a book value of $26,600. On the date of the exchange, the old equipment had a fair value of $28,000. In addition, Kelso paid $91,000 cash for the new equipment, which had a list price of $126,000. The exchange lacked commercial substance. At what amount should Kelso record the new equipment for financial accounting purposes? a. $91,000. b. $117,600. c. $119,000. d. $126,000.

Use the following information for questions 108 and 109.


Valuation of Inventories: A Cost-Basis Approach

8 - 115

A machine cost $360,000, has annual depreciation of $60,000, and has accumulated depreciation of $270,000 on December 31, 2012. On April 1, 2013, when the machine has a fair value of $82,500, it is exchanged for a machine with a fair value of $405,000 and the proper amount of cash is paid. The exchange lacked commercial substance. 108. The gain to be recorded on the exchange is a. $0. b. $7,500 loss. c. $15,000 gain. d. $45,000 gain. 109. The new machine should be recorded at a. $322,500. b. $367,500. c. $397,500. d. $405,000. Use the following information for questions 110 and 111. Equipment that cost $88,000 and has accumulated depreciation of $40,000 is exchanged for equipment with a fair value of $64,000 and $16,000 cash is received. The exchange lacked commercial substance. 110.

The gain to be recognized from the exchange is a. $6,400 gain. b. $8,000 gain. c. $24,000 gain. d. $32,000 gain.

111.

The new equipment should be recorded at a. $64,000. b. $48,000. c. $40,000. d. $38,400.

Use the following information for questions 112 through 114. Two independent companies, Hager Co. and Shaw Co., are in the home building business. Each owns a tract of land held for development, but each would prefer to build on the other's land. They agree to exchange their land. An appraiser was hired, and from her report and the companies' records, the following information was obtained: Hager's Land Shaw's Land Cost and book value $576,000 $360,000 Fair value based upon appraisal 720,000 630,000 The exchange was made, and based on the difference in appraised fair values, Shaw paid $90,000 to Hager. The exchange lacked commercial substance.


8 - 116

112.

Test Bank for Intermediate Accounting, Fourteenth Edition

For financial reporting purposes, Hager should recognize a pre-tax gain on this exchange of a. $0. b. $18,000. c. $90,000. d. $144,000.


Valuation of Inventories: A Cost-Basis Approach

8 - 117

113.

The new land should be recorded on Hager's books at a. $504,000. b. $576,000. c. $630,000. d. $720,000.

114.

The new land should be recorded on Shaw's books at a. $360,000. b. $450,000. c. $630,000. d. $720,000.

115.

Timmons Company traded machinery with a book value of $240,000 and a fair value of $400,000. It received in exchange from Lewis Company a machine with a fair value of $360,000 and cash of $40,000. Lewis’s machine has a book value of $380,000. What amount of gain should Timmons recognize on the exchange? a. $ -0b. $16,000 c. $40,000 d. $160,000

116.

Lewis Company traded machinery with a book value of $570,000 and a fair value of $540,000. It received in exchange from Timmons Company a machine with a fair value of $600,000. Lewis also paid cash of $60,000 in the exchange. Timmons’s machine has a book value of $570,000. What amount of gain or loss should Lewis recognize on the exchange? a. $60,000 gain b. $ -0-. c. $3,000 loss d. $30,000 loss

117.

Durler Company traded machinery with a book value of $540,000 and a fair value of $900,000. It received in exchange from Hoyle Company a machine with a fair value of $810,000 and cash of $90,000. Hoyle’s machine has a book value of $855,000. What amount of gain should Durler recognize on the exchange? a. $ -0b. $36,000 c. $90,000 d. $360,000

118.

Hoyle Company traded machinery with a book value of $570,000 and a fair value of $540,000. It received in exchange from Durler Company a machine with a fair value of $600,000. Hoyle also paid cash of $60,000 in the exchange. Durler’s machine has a book value of $570,000. What amount of gain or loss should Hoyle recognize on the exchange? a. $60,000 gain b. $ -0c. $3,000 loss


8 - 118

Test Bank for Intermediate Accounting, Fourteenth Edition

d. $30,000 loss


Valuation of Inventories: A Cost-Basis Approach

8 - 119

119.

Peterson Company purchased machinery for $480,000 on January 1, 2009. Straight-line depreciation has been recorded based on a $30,000 salvage value and a 5-year useful life. The machinery was sold on May 1, 2013 at a gain of $9,000. How much cash did Peterson receive from the sale of the machinery? a. $69,000 b. $81,000 c. $99,000 d. $129,000

120.

Sutherland Company purchased machinery for $640,000 on January 1, 2009. Straight-line depreciation has been recorded based on a $40,000 salvage value and a 5-year useful life. The machinery was sold on May 1, 2013 at a gain of $12,000. How much cash did Sutherland receive from the sale of the machinery? a. $92,000. b. $108,000. c. $132,000. d. $172,000.

121.

Ecker Company purchased a new machine on May 1, 2004 for $264,000. At the time of acquisition, the machine was estimated to have a useful life of ten years and an estimated salvage value of $12,000. The company has recorded monthly depreciation using the straight-line method. On March 1, 2013, the machine was sold for $36,000. What should be the loss recognized from the sale of the machine? a. $0. b. $5,400. c. $12,000. d. $17,400.

122.

On January 1, 2004, Mill Corporation purchased for $304,000, equipment having a useful life of ten years and an estimated salvage value of $16,000. Mill has recorded monthly depreciation of the equipment on the straight-line method. On December 31, 2012, the equipment was sold for $56,000. As a result of this sale, Mill should recognize a gain of a. $0. b. $11,200. c. $27,200. d. $56,000.

Multiple Choice Answers—Computational


8 - 120 Test Bank for Intermediate Accounting, Fourteenth Edition

MULTIPLE CHOICE—CPA Adapted 123.

On December 1, 2012, Hogan Co. purchased a tract of land as a factory site for $900,000. The old building on the property was razed, and salvaged materials resulting from demolition were sold. Additional costs incurred and salvage proceeds realized during December 2012 were as follows: Cost to raze old building Legal fees for purchase contract and to record ownership Title guarantee insurance Proceeds from sale of salvaged materials

$70,000 10,000 16,000 8,000

In Hogan 's December 31, 2012 balance sheet, what amount should be reported as land? a. $926,000. b. $962,000. c. $988,000. d. $996,000. 124.

Land was purchased to be used as the site for the construction of a plant. A building on the property was sold and removed by the buyer so that construction on the plant could begin. The proceeds from the sale of the building should be a. classified as other income. b. deducted from the cost of the land. c. netted against the costs to clear the land and expensed as incurred. d. netted against the costs to clear the land and amortized over the life of the plant.

125.

A company is constructing an asset for its own use. Construction began in 2012. The asset is being financed entirely with a specific new borrowing. Construction expenditures were made in 2012 and 2013 at the end of each quarter. The total amount of interest cost capitalized in 2013 should be determined by applying the interest rate on the specific new borrowing to the a. total accumulated expenditures for the asset in 2012 and 2013. b. average accumulated expenditures for the asset in 2012 and 2013. c. average expenditures for the asset in 2013. d. total expenditures for the asset in 2013.

126.

Colt Football Co. had a player contract with Watts that is recorded in its books at $4,800,000 on July 1, 2012. Day Football Co. had a player contract with Kurtz that is recorded in its books at $6,000,000 on July 1, 2012. On this date, Colt traded Watts to Day for Kurtz and paid a cash difference of $600,000. The fair value of the Kurtz contract was $7,200,000 on the exchange date. The exchange had no commercial substance. After the exchange, the Kurtz contract should be recorded in Colt's books at a. $5,400,000. b. $6,000,000. c. $6,600,000. d. $7,200,000.


Valuation of Inventories: A Cost-Basis Approach

8 - 121

127.

Huff Co. exchanged nonmonetary assets with Sayler Co. No cash was exchanged and the exchange had no commercial substance. The carrying amount of the asset surrendered by Huff exceeded both the fair value of the asset received and Sayler's carrying amount of that asset. Huff should recognize the difference between the carrying amount of the asset it surrendered and a. the fair value of the asset it received as a loss. b. the fair value of the asset it received as a gain. c. Sayler's carrying amount of the asset it received as a loss. d. Sayler's carrying amount of the asset it received as a gain.

128.

Chase County owned an idle parcel of real estate consisting of land and a factory building. Chase gave title to this realty to Patton Co. as an incentive for Patton to establish manufacturing operations in the County. Patton paid nothing for this realty, which had a fair market value of $250,000 at the date of the grant. Patton should record this nonmonetary transaction as a a. memo entry only. b. credit to Contribution Revenue for $250,000. c. credit to Extraordinary Income for $250,000. d. credit to Donated Capital for $250,000.

129.

On September 10, 2012, Jenks Co. incurred the following costs for one of its printing presses: Purchase of attachment $65,000 Installation of attachment 5,000 Replacement parts for renovation of press 18,000 Labor and overhead in connection with renovation of press 7,000 Neither the attachment nor the renovation increased the estimated useful life of the press. However, the renovation resulted in significantly increased productivity. What amount of the costs should be capitalized? a. $0. b. $77,000. c. $88,000. d. $95,000.

130.

On January 2, 2012, York Corp. replaced its boiler with a more efficient one. The following information was available on that date: Purchase price of new boiler Carrying amount of old boiler Fair value of old boiler Installation cost of new boiler

$170,000 10,000 4,000 20,000

The old boiler was sold for $4,000. What amount should York capitalize as the cost of the new boiler? a. $190,000. b. $186,000. c. $180,000. d. $170,000.


8 - 122 Test Bank for Intermediate Accounting, Fourteenth Edition


Valuation of Inventories: A Cost-Basis Approach

8 - 123

Multiple Choice Answers—CPA Adapted

IFRS QUESTIONS True/False 1. Under international accounting standards, historical cost is the preferred treatment for property, plant, and equipment. 2. Recently changes to IFRS require companies to capitalize borrowing costs related to qualifying assets. 3. Under IFRS, interest costs incurred during construction of a plant asset cannot be capitalized. 4. Under IFRS, if a company uses the revaluation model for fixed assets, companies must revalue the class of assets regularly. 5. Under IFRS, assets that qualify for interest capitalization are assets that are in use or ready for their intended use. Answers to True/False: Multiple Choice 1. Under IFRS, Sampson Company, who has a non-current asset which has been classified as held-for-sale, should a. test the asset's value monthly for impairment. b. value the asset at its depreciated historical cost. c. depreciate the asset over its remaining life. d. not depreciate the asset. 2. Miller Company, a company who uses IFRS reporting standards, sells a non-current asset classified as held-for-sale. Which of the following statements is true regarding the treatment of a gain on a subsequent increase in the fair value less cost? a. The gain should not be recognized. b. The gain should be recognized in full in the income statement. c. The gain should be recognized but only in retained earnings. d. The gain should be recognized to the extent that it is not in excess of the cumulative impairment loss that has been recognized.


8 - 124 Test Bank for Intermediate Accounting, Fourteenth Edition

3. Danson Company, a company who uses IFRS reporting standards, has a non-current asset that has been classified as held-for-sale. When the asset no longer meets this definition, Danson should a. remove the asset from the statement of financial position. b. remeasure the asset at fair value. c. measure the asset at the lower of its carrying value before it was classified as held-for-sale and its recoverable amount at the date when the company decided not to sell it. d. leave the non-current asset on the financial statements at the current carrying value. 4. Elton Industries, a company who uses IFRS reporting standards, has assets and liabilities of a disposal group classified as held-for-sale shown on its statement of financial position. Which of the following presents the best treatment for these? a. These assets and liabilities should be netted and presented as a single amount - either a current asset or a current liability on the statement of financial position. b. On the balance sheet, the disposal group assets should be shown separately from other assets, while the disposal group liabilities should be shown separately from other liabilities. c. The assets and liabilities should be netted and presented as a deduction from equity on the statement of financial position. d. There should be no separate disclosure of these assets and liabilities on the statement of financial position.


Valuation of Inventories: A Cost-Basis Approach

8 - 125

5. Woodson Company, a company who uses IFRS reporting standards, has identified a group of plant assets for disposal. On January 1, 2012, the carrying value of these assets was $17.5 million. The assets were revalued to $16.5 million on January 5, 2012, when they were identified as property for the disposal group. In addition, Woodson thinks that it will cost $1.5 million to sell these assets. What carrying amount should these assets reflect for year-end financial statements to be prepared on January 10, 2012? a. $17.5 million b. $16.5 million c. $16.0 million d. $15.0 million 6. Thomas Company, a company who uses IFRS reporting standards, is disposing of a plant asset. The amount of gain or loss from this disposal is a. reported as the difference between the sales proceeds and the carrying amount of the asset. b. not reported. c. reported as the fair value less the recoverable amount. d. reported as the difference between the net cash flows of the productive years of the asset and its carrying value. 7. On January 1, 2012, Jackson Company has a building with a carrying value of $50,000 and a remaining useful life 5 years that was recently valued at $150,000. Assuming that the company uses straight-line depreciation, IFRS would show the depreciation as a. $10,000 b. $30,000 c. $20,000 d. More than one of these answers could be correct. 8. Tram Industries, a company who uses IFRS reporting standards, is installing a new plant. The company has incurred the following costs 1. Operating losses before commercial production $ 200,000 2. Cost of the plant 1,500,000 3. Initial delivery and handling charges 300,000 4. Cost of site preparation 175,000 Which of these costs can Tram capitalize in accordance with IFRS? a. 1, 2, 3, & 4 b. 2 & 4 c. 2, 3, & 4 d. 1, 2, & 4


8 - 126 Test Bank for Intermediate Accounting, Fourteenth Edition

9. Icon Industries, a company who uses IFRS reporting standards, is installing a new plant. The company has incurred the following costs 1. Consultants used for advice on the acquisition of the plant $245,000 2. Interest charges paid to the supplier of plant for deferred credit $275,000 3. Estimated dismantling cost to be incurred after 8 years $400,000 4. Cost of the plant $2,300,000 Which of these costs can Tram capitalize in accordance with IFRS? a. 1, 2, 3, & 4 b. 4 only c. 1 & 4 d. 1, 3, & 4 10. All of the following are true regarding the revaluation model allowed under IFRS except a. once selected, the revaluation policy applies to an entire class of property, plant and equipment. b. revaluations must be made regularly to ensure that the carrying value is not materially different from fair value. c. after initial recognition, the revalued amount is fair value less subsequent depreciation and impairment losses. d. when an asset is revalued, any increase in carrying amount is reported as miscellaneous revenue. Answers to Multiple Choice: CHAPTER 11 DEPRECIATION, IMPAIRMENTS, AND DEPLETION IFRS questions are available at the end of this chapter. TRUE-FALSE—Conceptual 1.

Depreciation is a means of cost allocation, not a matter of valuation.

2.

Depreciation is based on the decline in the fair market value of the asset.

3.

Depreciation, depletion, and amortization all involve the allocation of the cost of a longlived asset to expense.

4.

The cost of an asset less its salvage value is its depreciation base.

5.

The three factors involved in the depreciation process are the depreciation base, the useful life, and the risk of obsolescence.


Valuation of Inventories: A Cost-Basis Approach

8 - 127

6.

Inadequacy is the replacement of one asset with another more efficient and economical asset.

7.

The major objection to the straight-line method is that it assumes the asset’s economic usefulness and repair expense are the same each year.

8.

The units-of-production approach to depreciation is appropriate when depreciation is a function of time instead of activity.

9.

An accelerated depreciation method is appropriate when the asset’s economic usefulness is the same each year.

10.

The declining-balance method does not deduct the salvage value in computing the depreciation base.

11.

Gains or losses on disposals of assets do not distort periodic income when the group or composite method is used to compute depreciation.

12.

Companies frequently use the composite approach when the assets are similar in nature and have approximately the same useful lives.

13.

Changes in estimates are handled prospectively by dividing the asset’s book value less any salvage value by the remaining estimated life.

14.

An impairment loss is the amount by which the carrying amount of the asset exceeds the sum of the expected future net cash flows from the use of that asset.

15.

The first step in determining whether an impairment has occurred is to estimate the future net cash flows expected from the use of that asset and its eventual disposition.

16.

Impaired assets held for disposal should be reported at the lower of cost or net realizable value.

17.

Normally, companies compute depletion on a straight-line basis.

18.

Intangible development costs and restoration costs are part of the depletion base.

19.

The asset turnover ratio is computed by dividing net sales by ending total assets.

20.

The profit margin on sales ratio is a measure for analyzing the use of property, plant, and equipment.

True False Answers—Conceptual


8 - 128 Test Bank for Intermediate Accounting, Fourteenth Edition

MULTIPLE CHOICE—Conceptual 21.

The following is true of depreciation accounting. a. It is not a matter of valuation. b. It is part of the matching of revenues and expenses. c. It retains funds by reducing income taxes and dividends. d. All of these.

22.

Which of the following principles best describes the conceptual rationale for the methods of matching depreciation expense with revenues? a. Associating cause and effect b. Systematic and rational allocation c. Immediate recognition d. Partial recognition

23.

Depreciation accounting a. provides funds. b. funds replacements. c. retains funds. d. all of these.

S

24.

Which of the following most accurately reflects the concept of depreciation as used in accounting? a. The process of charging the decline in value of an economic resource to income in the period in which the benefit occurred. b. The process of allocating the cost of tangible assets to expense in a systematic and rational manner to those periods expected to benefit from the use of the asset. c. A method of allocating asset cost to an expense account in a manner which closely matches the physical deterioration of the tangible asset involved. d. An accounting concept that allocates the portion of an asset used up during the year to the contra asset account for the purpose of properly recording the fair market value of tangible assets.


Valuation of Inventories: A Cost-Basis Approach

8 - 129

S

The major difference between the service life of an asset and its physical life is that a. service life refers to the time an asset will be used by a company and physical life refers to how long the asset will last. b. physical life is the life of an asset without consideration of salvage value and service life requires the use of salvage value. c. physical life is always longer than service life. d. service life refers to the length of time an asset is of use to its original owner, while physical life refers to how long the asset will be used by all owners.

P

26.

The term "depreciable base," or "depreciation base," as it is used in accounting, refers to a. the total amount to be charged (debited) to expense over an asset's useful life. b. the cost of the asset less the related depreciation recorded to date. c. the estimated market value of the asset at the end of its useful life. d. the acquisition cost of the asset.

27.

Economic factors that shorten the service life of an asset include a. obsolescence. b. supersession. c. inadequacy. d. all of these.

28.

Which of the following is not one of the basic questions that must be answered before the amount of depreciation charge can be computed? a. What is the depreciation base to use for the asset? b. What is the asset's useful life? c. What method of cost apportionment is best for this asset? d. What product or service is the asset related to?

25.

S

29.

Which of the following is a realistic assumption of the straight-line method of depreciation? a. The asset's economic usefulness is the same each year. b. The repair and maintenance expense is essentially the same each period. c. The rate of return analysis is enhanced using the straight-line method. d. Depreciation is a function of time rather than a function of usage.

30.

The activity method of depreciation a. is a variable charge approach. b. assumes that depreciation is a function of the passage of time. c. conceptually associates cost in terms of input measures. d. all of these.

31.

For income statement purposes, depreciation is a variable expense if the depreciation method used is a. units-of-production. b. straight-line. c. sum-of-the-years'-digits. d. declining-balance.


8 - 130 Test Bank for Intermediate Accounting, Fourteenth Edition


Valuation of Inventories: A Cost-Basis Approach

8 - 131

32.

If an industrial firm uses the units-of-production method for computing depreciation on its only plant asset, factory machinery, the credit to accumulated depreciation from period to period during the life of the firm will a. be constant. b. vary with unit sales. c. vary with sales revenue. d. vary with production.

33.

Use of the double-declining balance method a. results in a decreasing charge to depreciation expense. b. means salvage value is not deducted in computing the depreciation base. c. means the book value should not be reduced below salvage value. d. all of these.

34.

Use of the sum-of-the-years'-digits method a. results in salvage value being ignored. b. means the denominator is the years remaining at the beginning of the year. c. means the book value should not be reduced below salvage value. d. all of these.

35.

A graph is set up with "yearly depreciation expense" on the vertical axis and "time" on the horizontal axis. Assuming linear relationships, how would the graphs for straight-line and sum-of-the-years'-digits depreciation, respectively, be drawn? a. Vertically and sloping down to the right b. Vertically and sloping up to the right c. Horizontally and sloping down to the right d. Horizontally and sloping up to the right

36.

A principal objection to the straight-line method of depreciation is that it a. provides for the declining productivity of an aging asset. b. ignores variations in the rate of asset use. c. tends to result in a constant rate of return on a diminishing investment base. d. gives smaller periodic write-offs than decreasing charge methods.

37.

Each year a company has been investing an increasingly greater amount in machinery. Since there is a large number of small items with relatively similar useful lives, the company has been applying straight-line depreciation at a uniform rate to the machinery as a group. The ratio of this group's total accumulated depreciation to the total cost of the machinery has been steadily increasing and now stands at .75 to 1.00. The most likely explanation for this increasing ratio is the a. company should have been using one of the accelerated methods of depreciation. b. estimated average life of the machinery is less than the actual average useful life. c. estimated average life of the machinery is greater than the actual average useful life. d. company has been retiring fully depreciated machinery that should have remained in service.

38.

For the composite method, the composite


8 - 132 Test Bank for Intermediate Accounting, Fourteenth Edition

a. b. c. d.

rate is the total cost divided by the total annual depreciation. rate is the total annual depreciation divided by the total depreciable cost. life is the total cost divided by the total annual depreciation. life is the total depreciable cost divided by the total annual depreciation.

P

Watkins Truck Rental uses the group depreciation method for its fleet of trucks. When it retires one of its trucks and receives cash from a salvage company, the carrying value of property, plant, and equipment will be decreased by the a. original cost of the truck. b. original cost of the truck less the cash proceeds. c. cash proceeds received. d. cash proceeds received and original cost of the truck.

S

Composite or group depreciation is a depreciation system whereby a. the years of useful life of the various assets in the group are added together and the total divided by the number of items. b. the cost of individual units within an asset group is charged to expense in the year a unit is retired from service. c. a straight-line rate is computed by dividing the total of the annual depreciation expense for all assets in the group by the total cost of the assets. d. the original cost of all items in a given group or class of assets is retained in the asset account and the cost of replacements is charged to expense when they are acquired.

S

41.

When depreciation is computed for partial periods under a decreasing charge depreciation method, it is necessary to a. charge a full year's depreciation to the year of acquisition. b. determine depreciation expense for the full year and then prorate the expense between the two periods involved. c. use the straight-line method for the year in which the asset is sold or otherwise disposed of. d. use a salvage value equal to the first year's partial depreciation charge.

42.

Depreciation is normally computed on the basis of the nearest a. full month and to the nearest cent. b. full month and to the nearest dollar. c. day and to the nearest cent. d. day and to the nearest dollar.

43.

Myers Company acquired machinery on January 1, 2007 which it depreciated under the straight-line method with an estimated life of fifteen years and no salvage value. On January 1, 2012, Myers estimated that the remaining life of this machinery was six years with no salvage value. How should this change be accounted for by Myers? a. As a prior period adjustment b. As the cumulative effect of a change in accounting principle in 2012 c. By setting future annual depreciation equal to one-sixth of the book value on January 1, 2012 d. By continuing to depreciate the machinery over the original fifteen year life

39.

40.


Valuation of Inventories: A Cost-Basis Approach

44.

A change in estimate should a. result in restatement of prior period statements. b. be handled in current and future periods. c. be handled in future periods only. d. be handled retroactively.

8 - 133


8 - 134 Test Bank for Intermediate Accounting, Fourteenth Edition

45.

Lynch Printing Company determines that a printing press used in its operations has suffered a permanent impairment in value because of technological changes. An entry to record the impairment should a. recognize an extraordinary loss for the period. b. include a credit to the equipment accumulated depreciation account. c. include a credit to the equipment account. d. not be made if the equipment is still being used.

46.

Which of following is not a similarity in the accounting treatment for depreciation and cost depletion? a. The estimated life is based on economic or productive life. b. Assets subject to either are reported in the same classification on the balance sheet. c. The rates may be changed upon revision of the estimated productive life used in the original rate computations. d. Both depreciation and depletion are based on time.

47.

Which of the following is not a difference between the accounting treatment for depreciation and cost depletion? a. Depletion applies to natural resources while depreciation applies to plant and equipment. b. Depletion refers to the physical exhaustion or consumption of the asset while depreciation refers to the wear, tear, and obsolescence of the asset. c. Many formulas are used in computing depreciation but only one is used to any extent in computing depletion. d. The cost of the asset is the starting point from which computation of the amount of the periodic charge is made to operations for depreciation, but the fair value reassessed each year as the starting point for the periodic charge for depletion.

48.

Dividends representing a return of capital to stockholders are not uncommon among companies which a. use accelerated depreciation methods. b. use straight-line depreciation methods. c. recognize both functional and physical factors in depreciation. d. none of these.

49.

Depletion expense a. is usually part of cost of goods sold. b. includes tangible equipment costs in the depletion base. c. excludes intangible development costs from the depletion base. d. excludes restoration costs from the depletion base.

50.

The most common method of recording depletion for accounting purposes is the a. percentage depletion method. b. decreasing charge method. c. straight-line method. d. units-of-production method.


Valuation of Inventories: A Cost-Basis Approach

51.

8 - 135

Reserve recognition accounting a. is presently the generally accepted accounting method for financial reporting of oil and gas reserves. b. is a historical cost method similar to the full cost approach and the successful efforts approach. c. is used for reporting of oil and gas reserves for federal income tax purposes. d. requires estimates of future production costs, the appropriate discount rate, and the expected selling price of oil and gas reserves.

S

Of the following costs related to the development of natural resources, which one is not a part of depletion cost? a. Acquisition cost of the natural resource deposit b. Exploration costs c. Tangible equipment costs associated with machinery used to extract the natural resource d. Intangible development costs such as drilling costs, tunnels, and shafts

S

Which of the following disclosures is not required in the financial statements regarding depreciation? a. Accumulated depreciation, either by major classes of depreciable assets or in total. b. Details demonstrating how depreciation was calculated. c. Depreciation expense for the period. d. Balances of major classes of depreciable assets, by nature and function.

P

54.

The book value of a plant asset is a. the fair market value of the asset at a balance sheet date. b. the asset's acquisition cost less the total related depreciation recorded to date. c. equal to the balance of the related accumulated depreciation account. d. the assessed value of the asset for property tax purposes.

55.

A general description of the depreciation methods applicable to major classes of depreciable assets a. is not a current practice in financial reporting. b. is not essential to a fair presentation of financial position. c. is needed in financial reporting when company policy differs from income tax policy. d. should be included in corporate financial statements or notes thereto.

56.

The asset turnover ratio is computed by dividing a. net income by ending total assets. b. net income by average total assets. c. net sales by ending total assets. d. net sales by average total assets.

57.

The rate of return on total assets is computed by dividing a. Net income by ending total assets. b. Net sales by average total assets. c. Net sales by ending total assets.

52.

53.


8 - 136 Test Bank for Intermediate Accounting, Fourteenth Edition

d. Net income by average total assets.


Valuation of Inventories: A Cost-Basis Approach

8 - 137

*58.

A major objective of MACRS for tax depreciation is to a. reduce the amount of depreciation deduction on business firms' tax returns. b. assure that the amount of depreciation for tax and book purposes will be the same. c. help companies achieve a faster write-off of their capital assets. d. require companies to use the actual economic lives of assets in calculating tax depreciation.

*59.

Under MACRS, which one of the following is not considered in determining depreciation for tax purposes? a. Cost of asset b. Property recovery class c. Half-year convention d. Salvage value

*60.

If income tax effects are ignored, accelerated depreciation methods a. provide funds for the earlier replacement of fixed assets. b. increase funds provided by operations. c. tend to offset the effect of steadily increasing repair and maintenance costs on the income statement. d. tend to decrease the fixed asset turnover ratio.

Multiple Choice Answers—Conceptual Solutions to those Multiple Choice questions for which the answer is “none of these.” 48.

do not expect to purchase additional property after depleting existing property.

MULTIPLE CHOICE—Computational 61.

Ferguson Company purchased a depreciable asset for $120,000. The estimated salvage value is $10,000, and the estimated useful life is 10 years. The straight-line method will be used for depreciation. What is the depreciation base of this asset? a. $11,000 b. $12,000 c. $110,000 d. $120,000


8 - 138 Test Bank for Intermediate Accounting, Fourteenth Edition

62.

Hamilton Company purchased a depreciable asset for $240,000. The estimated salvage value is $20,000, and the estimated useful life is 10 years. The straight-line method will be used for depreciation. What is the depreciation base of this asset? a. $22,000 b. $24,000 c. $220,000 d. $240,000

63.

Solar Products purchased a machine for $39,000 on July 1, 2012. The company intends to depreciate it over 4 years using the double-declining balance method. Salvage value is $3,000. Depreciation for 2012 is a. $19,500 b. $9,750 c. $14,625 d. $9,000

64.

Solar Products purchased a machine for $39,000 on July 1, 2012. The company intends to depreciate it over 4 years using the double-declining balance method. Salvage value is $3,000. Depreciation for 2013 is a. $19,500 b. $9,750 c. $14,625 d. $9,000

65.

Gardner Corporation purchased a truck at the beginning of 2012 for $90,000. The truck is estimated to have a salvage value of $3,600 and a useful life of 120,000 miles. It was driven 18,000 miles in 2012 and 32,000 miles in 2013. What is the depreciation expense for 2012? a. $13,500 b. $12,960 c. $21,600 d. $36,000

66.

Gardner Corporation purchased a truck at the beginning of 2012 for $90,000. The truck is estimated to have a salvage value of $3,600 and a useful life of 120,000 miles. It was driven 18,000 miles in 2012 and 32,000 miles in 2013. What is the depreciation expense for 2013? a. $24,000 b. $36,000 c. $23,040 d. $38,400

67.

Kinder Company purchased a depreciable asset for $280,000. The estimated salvage value is $14,000, and the estimated useful life is 10,000 hours. Kinder used the asset for 1,100 hours in the current year. The activity method will be used for depreciation. What is the depreciation expense on this asset? a. $26,600


Valuation of Inventories: A Cost-Basis Approach

8 - 139

b. $29,260 c. $30,800 d. $266,000 68.

Jamar Company purchased a depreciable asset for $225,000. The estimated salvage value is $15,000, and the estimated useful life is 8 years. The double-declining balance method will be used for depreciation. What is the depreciation expense for the second year on this asset? a. $26,250 b. $39,375 c. $42,188 d. $56,250

69.

Engels Company purchased a depreciable asset for $800,000. The estimated salvage value is $40,000, and the estimated useful life is 10,000 hours. Engels used the asset for 1,100 hours in the current year. The activity method will be used for depreciation. What is the depreciation expense on this asset? a. $76,000 b. $83,600 c. $88,000 d. $760,000

70.

Hart Company purchased a depreciable asset for $450,000. The estimated salvage value is $30,000, and the estimated useful life is 8 years. The double-declining balance method will be used for depreciation. What is the depreciation expense for the second year on this asset? a. $52,500 b. $78,750 c. $84,375 d. $112,500

71.

On July 1, 2012, Gonzalez Corporation purchased factory equipment for $225,000. Salvage value was estimated to be $6,000. The equipment will be depreciated over ten years using the double-declining balance method. Counting the year of acquisition as one-half year, Gonzalez should record depreciation expense for 2013 on this equipment of a. $45,000. b. $40,500. c. $39,420. d. $36,000.

72.

Krause Corporation purchased factory equipment that was installed and put into service January 2, 2012, at a total cost of $120,000. Salvage value was estimated at $8,000. The equipment is being depreciated over four years using the double-declining balance method. For the year 2013, Krause should record depreciation expense on this equipment of a. $28,000. b. $30,000. c. $56,000.


8 - 140 Test Bank for Intermediate Accounting, Fourteenth Edition

d. $60,000. 73.

74.

On April 13, 2012, Neill Co. purchased machinery for $168,000. Salvage value was estimated to be $7,000. The machinery will be depreciated over ten years using the doubledeclining balance method. If depreciation is computed on the basis of the nearest full month, Neill should record depreciation expense for 2013 on this machinery of a. $29,120. b. $28,560. c. $28,770. d. $29,306. Matile Co. purchased machinery that was installed and ready for use on January 3, 2012, at a total cost of $115,000. Salvage value was estimated at $15,000. The machinery will be depreciated over five years using the double-declining balance method. For the year 2013, Matile should record depreciation expense on this machinery of a. $24,000. b. $27,600. c. $30,000. d. $46,000.

75.

A plant asset has a cost of $32,000 and a salvage value of $8,000. The asset has a threeyear life. If depreciation in the third year amounted to $4,000, which depreciation method was used? a. Straight-line b. Declining-balance c. Sum-of-the-years'-digits d. Cannot tell from information given

76.

On January 1, 2012, Graham Company purchased a new machine for $2,800,000. The new machine has an estimated useful life of nine years and the salvage value was estimated to be $100,000. Depreciation was computed on the sum-of-the-years'-digits method. What amount should be shown in Graham's balance sheet at December 31, 2013, net of accumulated depreciation, for this machine? a. $2,260,000 b. $1,780,000 c. $1,742,221 d. $1,659,000

77.

On January 1, 2006, Forbes Company purchased equipment at a cost of $100,000. The equipment was estimated to have a salvage value of $10,000 and it is being depreciated over eight years under the sum-of-the-years'-digits method. What should be the charge for depreciation of this equipment for the year ended December 31, 2013? a. $2,500 b. $2,778 c. $5,000 d. $11,250


Valuation of Inventories: A Cost-Basis Approach

78.

8 - 141

On September 19, 2012, McCoy Co. purchased machinery for $285,000. Salvage value was estimated to be $15,000. The machinery will be depreciated over eight years using the sumof-the-years'-digits method. If depreciation is computed on the basis of the nearest full month, McCoy should record depreciation expense for 2013 on this machinery of a. $61,354. b. $58,267. c. $58,125. d. $52,500.


8 - 142 Test Bank for Intermediate Accounting, Fourteenth Edition

79.

On January 3, 2011, Munoz Co. purchased machinery. The machinery has an estimated useful life of eight years and an estimated salvage value of $60,000. The depreciation applicable to this machinery was $130,000 for 2013, computed by the sum-of-the-years'digits method. The acquisition cost of the machinery was a. $720,000. b. $780,000. c. $840,000. d. $936,000.

80.

On January 2, 2010, Stacy Company acquired equipment to be used in its manufacturing operations. The equipment has an estimated useful life of 10 years and an estimated salvage value of $30,000. The depreciation applicable to this equipment was $140,000 for 2013, computed under the sum-of-the-years'-digits method. What was the acquisition cost of the equipment? a. $1,070,000 b. $1,130,000 c. $1,100,000 d. $1,083,333

81.

Orton Corporation, which has a calendar year accounting period, purchased a new machine for $60,000 on April 1, 2008. At that time Orton expected to use the machine for nine years and then sell it for $6,000. The machine was sold for $33,000 on Sept. 30, 2013. Assuming straight-line depreciation, no depreciation in the year of acquisition, and a full year of depreciation in the year of retirement, the gain to be recognized at the time of sale would be a. $6,000. b. $4,500. c. $3,000. d. $0.

82.

On January 1, 2012, the Accumulated Depreciation—Machinery account of a particular company showed a balance of $740,000. At the end of 2012, after the adjusting entries were posted, it showed a balance of $790,000. During 2012, one of the machines which cost $250,000 was sold for $121,000 cash. This resulted in a loss of $8,000. Assuming that no other assets were disposed of during the year, how much was depreciation expense for 2012? a. $171,000 b. $187,000 c. $50,000 d. $121,000

83.

During 2012, Noller Co. sold equipment that had cost $294,000 for $176,400. This resulted in a gain of $12,900. The balance in Accumulated Depreciation—Equipment was $975,000 on January 1, 2012, and $930,000 on December 31. No other equipment was disposed of during 2012. Depreciation expense for 2012 was a. $45,000. b. $57,900.


Valuation of Inventories: A Cost-Basis Approach

c. $85,500. d. $175,500.

8 - 143


8 - 144 Test Bank for Intermediate Accounting, Fourteenth Edition

Use the following information for questions 84 and 85: A schedule of machinery owned by Mallon Co. is presented below: Estimated Estimated Total Cost Salvage Value Life in Years Machine A $260,000 $20,000 12 Machine C 390,000 30,000 10 Machine M 195,000 15,000 6 Mallon computes depreciation by the composite method. 84.

The composite rate of depreciation (in percent) for these assets is a. 10.18. b. 10.77. c. 11.03. d. 11.67.

85.

The composite life (in years) for these assets is a. 9.1. b. 9.3. c. 9.8. d. 10.0.

86.

Stevenson Company purchased a depreciable asset for $350,000 on April 1, 2010. The estimated salvage value is $35,000, and the estimated useful life is 5 years. The straightline method is used for depreciation. What is the balance in accumulated depreciation on May 1, 2013 when the asset is sold? a. $126,000 b. $147,000 c. $173,250 d. $194,250

87.

Williamson Corporation purchased a depreciable asset for $400,000 on January 1, 2010. The estimated salvage value is $40,000, and the estimated useful life is 9 years. The straight-line method is used for depreciation. In 2013, Williamson changed its estimates to a total useful life of 5 years with a salvage value of $60,000. What is 2013 depreciation expense? a. $40,000 b. $60,000 c. $110,000 d. $120,000

88.

Rollins Company purchased a depreciable asset for $500,000 on April 1, 2010. The estimated salvage value is $50,000, and the estimated total useful life is 5 years. The straight-line method is used for depreciation. What is the balance in accumulated depreciation on May 1, 2013 when the asset is sold? a. $196,667 b. $210,000


Valuation of Inventories: A Cost-Basis Approach

8 - 145

c. $247,500 d. $277,500 89.

Fanestil Corporation purchased a depreciable asset for $630,000 on January 1, 2010. The estimated salvage value is $63,000, and the estimated total useful life is 9 years. The straight-line method is used for depreciation. In 2013, Fanestill changed its estimates to a useful life of 5 years with a salvage value of $105,000. What is 2013 depreciation expense? a. $63,000 b. $105,000 c. $168,000 d. $189,000

90.

If Lawson, Inc. uses the composite method and its composite rate is 7.5% per year, what entry should it make when plant assets that originally cost $80,000 and have been used for 10 years are sold for $24,000? a. Cash 24,000 Accumulated Depreciation - Plant Assets 56,000 Plant Assets 80,000 b. Cash 24,000 Loss on Sale of Plant Assets 56,000 Plant Assets 80,000 c. Cash 24,000 Accumulated Depreciation - Plant Assets 60,000 Plant Assets 80,000 Gain on Sale of Plant Assets 4,000 d. Cash 24,000 Plant Assets 24,000

91.

Archer Company purchased equipment in January of 2002 for $150,000. The equipment was being depreciated on the straight-line method over an estimated useful life of 20 years, with no salvage value. At the beginning of 2012, when the equipment had been in use for 10 years, the company paid $25,000 to overhaul the equipment. As a result of this improvement, the company estimated that the useful life of the equipment would be extended an additional 5 years. What should be the depreciation expense recorded for this equipment in 2012. a. $5,000 b. $6,667 c. $7,500 d. $9,167

Use the following information to answer questions 92 and 93. Ebert Inc. owns the following assets: Asset

Cost

Salvage

Estimated Useful Life


8 - 146 Test Bank for Intermediate Accounting, Fourteenth Edition

A B C 92.

93.

$140,000 75,000 164,000

$14,000 7,500 8,000

10 years 5 years 12 years

What is the composite depreciation rate of Ebert's assets? a. 14.0% b. 10.3% c. 12.9% d. 11.1% What is the composite life of Ebert's assets? a. 14.0 years b. 9.7 years c. 8.9 years d. 10.3 years

94.

Technique Co. has equipment with a carrying amount of $1,600,000. The expected future net cash flows from the equipment are $1,630,000, and its fair value is $1,360,000. The equipment is expected to be used in operations in the future. What amount (if any) should Technique report as an impairment to its equipment? a. No impairment should be reported. b. $240,000 c. $30,000 d. $270,000

95.

Robertson Inc. bought a machine on January 1, 2002 for $400,000. The machine had an expected life of 20 years and was expected to have a salvage value of $40,000. On July 1, 2012, the company reviewed the potential of the machine and determined that its undiscounted future net cash flows totaled $200,000 and its discounted future net cash flows totaled $140,000. If no active market exists for the machine and the company does not plan to dispose of it, what should Robertson record as an impairment loss on July 1, 2012? a. $ 0 b. $11,000 c. $20,000 d. $71,000

96.

Holcomb Corpsssoration owns machinery with a book value of $285,000. It is estimated that the machinery will generate future cash flows of $300,000. The machinery has a fair value of $210,000. Holcomb should recognize a loss on impairment of a. $ -0-. b. $15,000. c. $75,000. d. $90,000.

97.

Kohlman Corporation owns machinery with a book value of $380,000. It is estimated that the machinery will generate future cash flows of $350,000. The machinery has a fair value of $280,000. Kohlman should recognize a loss on impairment of


Valuation of Inventories: A Cost-Basis Approach

a. b. c. d. 98.

99.

8 - 147

$ -0-. $ 30,000. $100,000. $ 70,000.

Marsh Corporation purchased a machine on July 1, 2010, for $1,250,000. The machine was estimated to have a useful life of 10 years with an estimated salvage value of $70,000. During 2013, it became apparent that the machine would become uneconomical after December 31, 2017, and that the machine would have no scrap value. Accumulated depreciation on this machine as of December 31, 2012, was $295,000. What should be the charge for depreciation in 2013 under generally accepted accounting principles? a. $177,000 b. $191,000 c. $205,000 d. $238,750 Rivera Company purchased a tooling machine on January 3, 2006 for $700,000. The machine was being depreciated on the straight-line method over an estimated useful life of 10 years, with no salvage value. At the beginning of 2013, the company paid $175,000 to overhaul the machine. As a result of this improvement, the company estimated that the useful life of the machine would be extended an additional 5 years (15 years total). What should be the depreciation expense recorded for the machine in 2013? a. $48,125 b. $58,333 c. $70,000 d. $77,000

100.

Gates Co. purchased machinery on January 2, 2007, for $660,000. The straight-line method is used and useful life is estimated to be 10 years, with a $60,000 salvage value. At the beginning of 2013 Gates spent $144,000 to overhaul the machinery. After the overhaul, Gates estimated that the useful life would be extended 4 years (14 years total), and the salvage value would be $30,000. The depreciation expense for 2013 should be a. $42,375. b. $51,750. c. $60,000. d. $55,500.

101.

Newell, Inc. purchased equipment in 2011 at a cost of $800,000. Two years later it became apparent to Newell, Inc. that this equipment had suffered an impairment of value. In early 2013, the book value of the asset is $480,000 and it is estimated that the fair value is now only $320,000. The entry to record the impairment is a. No entry is necessary as a write-off violates the historical cost principle. b. Retained Earnings............................................................ 160,000 Accumulated Depreciation—Equipment............. 160,000 c. Loss on Impairment of Equipment.................................. 160,000 Accumulated Depreciation—Equipment............. 160,000 d. Retained Earnings............................................................ 160,000 Reserve for Loss on Impairment of Equipment... 160,000


8 - 148 Test Bank for Intermediate Accounting, Fourteenth Edition

102.

Percy Resources Company acquired a tract of land containing an extractable natural resource. Percy is required by its purchase contract to restore the land to a condition suitable for recreational use after it has extracted the natural resource. Geological surveys estimate that the recoverable reserves will be 2,000,000 tons, and that the land will have a value of $1,000,000 after restoration. Relevant cost information follows: Land Estimated restoration costs

$7,500,000 1,500,000

If Percy maintains no inventories of extracted material, what should be the charge to depletion expense per ton of extracted material? a. $3.25 b. $3.75 c. $4.00 d. $4.50


Valuation of Inventories: A Cost-Basis Approach

8 - 149

103.

In January, 2012, Yoder Corporation purchased a mineral mine for $5,100,000 with removable ore estimated by geological surveys at 2,000,000 tons. The property has an estimated value of $300,000 after the ore has been extracted. The company incurred $1,500,000 of development costs preparing the mine for production. During 2012, 500,000 tons were removed and 400,000 tons were sold. What is the amount of depletion that Yoder should expense for 2012? a. $960,000 b. $1,200,000 c. $1,260,000 d. $1,680,000

104.

During 2012, Eldred Corporation acquired a mineral mine for $3,000,000 of which $400,000 was ascribed to land value after the mineral has been removed. Geological surveys have indicated that 10 million units of the mineral could be extracted. During 2012, 1,500,000 units were extracted and 1,200,000 units were sold. What is the amount of depletion expensed for 2012? a. $260,000. b. $312,000. c. $360,000. d. $390,000.

105.

In March, 2012, Maley Mines Co. purchased a coal mine for $8,000,000. Removable coal is estimated at 1,500,000 tons. Maley is required to restore the land at an estimated cost of $960,000, and the land should have a value of $840,000. The company incurred $2,000,000 of development costs preparing the mine for production. During 2012, 450,000 tons were removed and 300,000 tons were sold. The total amount of depletion that Maley should record for 2012 is a. $1,832,000. b. $2,024,000. c. $2,748,000. d. $3,036,000.

106.

In 2004, Horton Company purchased a tract of land as a possible future plant site. In January, 2012, valuable sulphur deposits were discovered on adjoining property and Horton Company immediately began explorations on its property. In December, 2012, after incurring $800,000 in exploration costs, which were accumulated in an expense account, Horton discovered sulphur deposits appraised at $4,500,000 more than the value of the land. To record the discovery of the deposits, Horton should a. make no entry. b. debit $800,000 to an asset account. c. debit $4,500,000 to an asset account. d. debit $5,300,000 to an asset account.

107.

Balcom Corporation acquires a coal mine at a cost of $1,500,000. Intangible development costs total $360,000. After extraction has occurred, Balcom must restore the property (estimated fair value of the obligation is $180,000), after which it can be sold for $510,000.


8 - 150 Test Bank for Intermediate Accounting, Fourteenth Edition

108.

Balcom estimates that 5,000 tons of coal can be extracted. What is the amount of depletion per ton? a. $306 b. $510 c. $300 d. $372 Balcom Corporation acquires a coal mine at a cost of $1,500,000. Intangible development costs total $360,000. After extraction has occurred, Balcom must restore the property (estimated fair value of the obligation is $180,000), after which it can be sold for $510,000. Balcom estimates that 5,000 tons of coal can be extracted. If 900 tons are extracted the first year, which of the following would be included in the journal entry to record depletion? a. Debit to Accumulated Depletion for $275,400 b. Debit to Inventory for $275,400 c. Credit to Inventory for $270,000 d. Credit to Accumulated Depletion for $459,000

109.

In 2012, MegaStores reported net income of $5.7 billion, net sales of $164.7 billion, and average total assets of $61.0 billion. What is MegaStores' asset turnover ratio? a. 0.37 times b. 0.09 times. c. 2.7 times. d. 10.7 times.

110.

In 2012, MegaStores reported net income of $5.7 billion, net sales of $164.7 billion, and average total assets of $61.0 billion. What is MegaStores' return on total assets? a. 9.3% b. 10.7% c. 37.0% d. 270%

Use the following information for questions 111 and 112: For 2012, Hoyle Company reports beginning of the year total assets of $900,000, end of the year total assets of $1,100,000, net sales of $750,000, and net income of $150,000. 111.

Hoyle’s 2012 asset turnover ratio is a. 0.14 times. b. 0.15 times. c. 0.68 times. d. 0.75 times.

112.

The rate of return on assets for Hoyle in 2012 is a. 12.0%. b. 13.6%. c. 15.0%. d. 16.7%.


Valuation of Inventories: A Cost-Basis Approach

113.

Markowitz Company reported the following data: Sales Net Income Assets at year end Liabilities at year end

114.

8 - 151

2012 $3,000,000 300,000 1,800,000 1,100,000

2013 $3,900,000 400,000 2,500,000 1,500,000

What is Markowitz’s asset turnover for 2013? a. 1.56 b. 1.61 c. 1.81 d. 2.17 Froelich Company reported the following data: Sales Net Income Assets at year end Liabilities at year end

2012 $3,000,000 300,000 1,800,000 1,100,000

2013 $4,200,000 400,000 2,500,000 1,500,000

What is Froelich’s asset turnover for 2013? a. 1.68 b. 1.72 c. 1.95 d. 2.33 Use the following information for questions 115 and 116: On January 1, 2012, Guzman Company purchased a machine costing $250,000. The machine is in the MACRS 5-year recovery class for tax purposes and has an estimated $50,000 salvage value at the end of its economic life. *115. Assuming the company uses the general MACRS approach, the amount of MACRS deduction for tax purposes for the year 2012 is a. $50,000. b. $100,000. c. $80,000. d. $40,000. *116. Assuming the company uses the optional straight-line method, the amount of MACRS deduction for tax purposes for the year 2012 is a. $40,000. b. $50,000. c. $20,000. d. $25,000.


8 - 152 Test Bank for Intermediate Accounting, Fourteenth Edition

Multiple Choice Answers—Computational

MULTIPLE CHOICE—CPA Adapted 117.

Pike Co. purchased a machine on July 1, 2012, for $800,000. The machine has an estimated useful life of five years and a salvage value of $160,000. The machine is being depreciated from the date of acquisition by the 150% declining-balance method. For the year ended December 31, 2012, Pike should record depreciation expense on this machine of a. $240,000. b. $160,000. c. $120,000. d. $96,000.

118.

A machine with a five-year estimated useful life and an estimated 10% salvage value was acquired on January 1, 2011. The depreciation expense for 2013 using the double-declining balance method would be original cost multiplied by a. 90% × 40% × 40%. b. 60% × 60% × 40%. c. 90% × 60% × 40%. d. 40% × 40%.

119.

On April 1, 2011, Verlin Co. purchased new machinery for $300,000. The machinery has an estimated useful life of five years, and depreciation is computed by the sum-of-theyears'-digits method. The accumulated depreciation on this machinery at March 31, 2013, should be a. $200,000. b. $180,000. c. $120,000. d. $100,000.

120.

Hahn Co. takes a full year's depreciation expense in the year of an asset's acquisition and no depreciation expense in the year of disposition. Data relating to one of Hahn's depreciable assets at December 31, 2013 are as follows: Acquisition year Cost Residual value Accumulated depreciation Estimated useful life

2011 $210,000 30,000 144,000 5 years

Using the same depreciation method as used in 2011, 2012, and 2013, how much depreciation expense should Hahn record in 2014 for this asset? a. $24,000 b. $36,000


Valuation of Inventories: A Cost-Basis Approach

c. $42,000 d. $48,000

8 - 153


8 - 154 Test Bank for Intermediate Accounting, Fourteenth Edition

121.

A depreciable asset has an estimated 15% salvage value. At the end of its estimated useful life, the accumulated depreciation would equal the original cost of the asset under which of the following depreciation methods? Straight-line Productive Output a. Yes No b. Yes Yes c. No Yes d. No No

122.

Net income is understated if, in the first year, estimated salvage value is excluded from the depreciation computation when using the Straight-line Production or Method Use Method a. Yes No b. Yes Yes c. No No d. No Yes

123.

A plant asset with a five-year estimated useful life and no residual value is sold at the end of the second year of its useful life. How would using the sum-of-the-years'-digits method of depreciation instead of the double-declining balance method of depreciation affect a gain or loss on the sale of the plant asset? Gain Loss a. Decrease Decrease b. Decrease Increase c. Increase Decrease d. Increase Increase

124.

Giger Company acquired a tract of land containing an extractable natural resource. Giger is required by the purchase contract to restore the land to a condition suitable for recreational use after it has extracted the natural resource. Geological surveys estimate that the recoverable reserves will be 5,000,000 tons, and that the land will have a value of $800,000 after restoration. Relevant cost information follows: Land Estimated restoration costs

$5,600,000 1,200,000

If Giger maintains no inventories of extracted material, what should be the charge to depletion expense per ton of extracted material? a. b. c. d.

$1.36 $1.20 $1.12 $0.96


Valuation of Inventories: A Cost-Basis Approach

125.

8 - 155

In January 2012, Fehr Mining Corporation purchased a mineral mine for $6,300,000 with removable ore estimated by geological surveys at 2,500,000 tons. The property has an estimated value of $600,000 after the ore has been extracted. Fehr incurred $1,725,000 of development costs preparing the property for the extraction of ore. During 2012, 340,000 tons were removed and 300,000 tons were sold. For the year ended December 31, 2012, Fehr should include what amount of depletion in its cost of goods sold? a. $775,200 b. $684,000 c. $891,000 d. $1,009,800

Multiple Choice Answers—CPA Adapted

IFRS QUESTIONS True / False 1.

Under both IFRS and U.S. GAAP, interest costs incurred during construction must be capitalized.

2.

As with U.S. GAAP, IFRS requires that both direct and indirect costs in self-constructed assets be capitalized.

3.

IFRS, like U.S. GAAP, capitalizes all direct costs in self-constructed assets.

4.

Even though IFRS does not employ the first-stage recoverability test used under U.S. GAAP  comparing the undiscounted cash flows to the carrying amount, the fact that IFRS uses a fair value test to measure impairment loss makes IFRS stricter than U.S. GAAP

5.

U.S. GAAP, like IFRS permits write-up for subsequent recoveries of impairment, back up to the original amount before the impairment in all circumstances.

6.

Unlike U.S. GAAP, interest costs incurred during construction are not capitalized under IFRS.

7.

Asset revaluations are permitted under IFRS and U.S. GAAP.

8.

In general, IFRS adheres to very different principles than U.S. GAAP.

9.

U.S. GAAP, per SFAS No. 153, now requires that gains on exchanges of nonmonetary assets be recognized if the exchange lacks commercial substance.


8 - 156 Test Bank for Intermediate Accounting, Fourteenth Edition

10.

IFRS permits the same depreciation methods as U.S GAAP, with the exception of the units-of-production method, which is not allowed under IFRS.

Answers to True / False questions


Valuation of Inventories: A Cost-Basis Approach

8 - 157

Multiple-Choice Questions 1.

IFRS uses a fair value test to measure impairment loss. However, IFRS does not use the first-stage recoverability test under U.S. GAAP  comparing the undiscounted cash flow to the carrying amount. As a result, the IFRS test is a. not as strict as U.S. GAAP. b. more strict than U.S. GAAP. c. essentially the same strictness as U.S. GAAP. d. None of the above.

2.

Acceptable depreciation methods under IFRS include a. Straight-line. b. Accelerated. c. Units-of-production. d. All of the above.

3.

The primary IFRS related to property, plant and equipment is found in a. IAS 1 and IAS 34. b. IAS 11 and IAS 17. c. IAS 16 and IAS 23. d. IAS 27 and IAS 39.

4.

The accounting exchanges of nonmonetary assets has recently converged between IFRS and U.S. GAAP, per SFAS No. 153, now requires a. that gains on exchanges of nonmonetary assets be recognized if the exchange has commercial substance. b. that gains on exchanges of nonmonetary assets be recognized if the exchange does not have commercial substance. c. that gains on exchanges of nonmonetary assets be recognized if the exchange does not have commercial substance, and has never been impaired. d. All of the above. 5.

In measuring an impairment loss, IFRS uses a. undiscounted cash flows. b. discounted cash flows. c. a fair value test. d. a replacement value test.

6.

IFRS permits companies to carry assets at historical cost or use a revaluation model for fixed assets. According to IAS 16, if revaluation is used: 1. it must be applied to all assets in a class of assets. 2. assets must be revalued on an annual basis. 3. assets must be depreciated on the straight-line basis. 4. salvage values must be zero. a. 1 is correct b. 2 is correct c. 1 and 2 are correct


8 - 158 Test Bank for Intermediate Accounting, Fourteenth Edition

d. All are correct


Valuation of Inventories: A Cost-Basis Approach

8 - 159

Questions 7 through 10 are based on the following information: Simpson Company applies revaluation accounting to plant assets with a carrying value of $1,600,000, a useful life of 4 years, and no salvage value. Depreciation is calculated on the straight-line basis. At the end of year 1, independent appraisers determine that the asset has a fair value of $1,500,000. 7.

The journal entry to record depreciation for year one will include a a. debit to Accumulated Depreciation for $400,000. b. debit to Depreciation Expense for $100,000. c. credit to Accumulated Depreciation for $100,000. d. debit to Depreciation Expense for $400,000.

8.

The journal entry to adjust the plant assets to fair value and record revaluation surplus in year one will include a a. debit to Accumulated Depreciation for $100,000. b. credit to Depreciation Expense for $300,000. c. credit to Plant Assets for $300,000. d. credit to Revaluation Surplus for $300,000.

9.

The financial statements for year one will include the following information a. Accumulated depreciation $400,000. b. Depreciation expense $100,000. c. Plant assets $1,500,000. d. Revaluation surplus $100,000.

10.

The entry to record depreciation for this same asset in year two will include a a. debit to Accumulated Depreciation for $400,000. b. debit to Depreciation Expense for $500,000. c. credit to Accumulated Depreciation for $300,000. d. debit to Depreciation Expense for $400,000.

Answers to multiple choice:


8 - 160 Test Bank for Intermediate Accounting, Fourteenth Edition

Short Answer: 1. Briefly describe some of the similarities and differences between U.S. GAAP and IFRS with respect to the accounting for property, plant, and equipment. 2.

At a recent executive committee meeting, the controller for Marino Company remarked, “With only a single key difference between U.S. GAAP and IFRS for property, plant, and equipment, it should be smooth sailing for the FASB and IASB to converge their standards in this area.� Prepare a response to the controller.


Valuation of Inventories: A Cost-Basis Approach

8 - 161

IFRS QUESTIONS True / False 1.

Who owns the goods, as well as the costs to include in inventory, are essentially accounted for the same under IFRS and U.S. GAAP.

2.

U.S. GAAP has less detailed rules related to the accounting for inventories, compared to IFRS.

3.

IFRS does not permit the LIFO method to account for inventories.

4.

Many U.S. companies that have international operations use LIFO for U.S. purposes but use FIFO for their foreign subsidiaries.

5.

Both U.S. GAAP and IFRS permit the use of the LIFO method to account for inventories.

Answers to True / False questions:

Multiple Choice Questions: 1.

Under IFRS, an entity should initially recognize inventory when a. it has control of the inventory b. it expects it to provide future economic benefits c. the cost of the inventory can be reliably measured d. all of these choices are correct

2.

With respect to accounting for inventories, which of the following is a difference that exists for IFRS, as opposed to U.S. GAAP? a. There is required recognition of certain development costs. b. The FIFO method of inventories is prohibited. c. The specific identification method of inventories is only allowed when goods are interchangeable. d. The weighted average method of inventories is prohibited.

3.

Under IFRS, which of the following would be included in the cost of inventories? a. Product specific designer costs b. Abnormal waste materials c. Selling costs d. All of these would be included in the cost of inventories.

4.

Which of the following best describes the IFRS requirement for applying the same cost formula to all inventories?


8 - 162 Test Bank for Intermediate Accounting, Fourteenth Edition

a. b. c. d. 5.

When they are purchased from different suppliers. When they are purchased from the same geographic region. When they are similar in nature or use. When they sell for the same price.

Under IFRS, inventories are classified as a. noncurrent assets b. current assets c. stockholders' equity d. current liabilities

Use the following information to answer questions 6-8. Barton Company uses a periodic inventory system. On January 1, 2012, Barton Company had 600 units of inventory on hand at a cost of $8 per unit. During 2012, Barton made the following inventory purchases. April 1 June 1 September 1 November 1

Purchased 200 units at $10 Purchased 150 units at $12 Purchased 400 units at $14 Purchased 500 units at $15

Assume Barton Company sold 1,150 units of inventory during 2012. 6.

If you assume that Barton follows IFRS and uses the FIFO method, what is the ending inventory and cost of goods sold, respectively? a. Ending inventory = $5,800; Cost of Goods Sold = $15,900 b. Ending inventory = $8,260; Cost of Goods Sold = $13,440 c. Ending inventory = $8,211; Cost of Goods Sold = $13,489 d. Ending inventory = $10,300; Cost of Goods Sold = $11,400

7.

If you assume that Barton follows IFRS and uses the Average-cost method, what is the ending inventory and cost of goods sold, respectively? a. Ending inventory = $5,800; Cost of Goods Sold = $15,900 b. Ending inventory = $8,260; Cost of Goods Sold = $13,440 c. Ending inventory = $8,211; Cost of Goods Sold = $13,489 d. Ending inventory = $10,300; Cost of Goods Sold = $11,400


Valuation of Inventories: A Cost-Basis Approach

8 - 163

8.

Based on your answers to Questions 6 and 7, which of the following is a disadvantage of using the IFRS FIFO method, as compared to Average-cost under U.S. GAAP? a. Under FIFO, during periods of inflation, inventory costs matched against sales are greater than the inventory replacement cost. b. When price levels increase and inventory quantities do not decrease, taxes are greater under FIFO c. FIFO may cause poorer buying habits as management attempts to manipulate net income. d. FIFO typically causes lower reported earnings.

9.

Which of the following is an advantage for U.S. companies with international operations to use LIFO for U.S. purposes, as opposed to using FIFO for foreign subsidiaries? a. LIFO creates paper profits. b. LIFO generally approximates the physical flow of items. c. Under LIFO, inventory is less vulnerable to price declines. d. LIFO eliminates balance sheet distortion.

10.

Both U.S. GAAP and IFRS exclude which of the following from the cost of inventory? a. Selling costs b. General administrative costs c. Most storage costs d. All of these are excluded by U.S. GAAP and IFRS.

Answer to Multiple Choice.


Turn static files into dynamic content formats.

Create a flipbook
Issuu converts static files into: digital portfolios, online yearbooks, online catalogs, digital photo albums and more. Sign up and create your flipbook.